You are on page 1of 73

CHAPTER 1 QUESTIONS

Chapter 1 Multiple-Choice Questions from Textbook


1. Which of the following individuals is considered to be the “Father of Microbiology”?

a. Anton van Leeuwenhoek


b. John Tyndall
c. Louis Pasteur
d. Robert Koch
e. Rudolf Virchow

2. The microorganisms that usually live on or within a person are collectively referred to as

a. germs
b. indigenous microflora
c. nonpathogens
d. opportunistic pathogens
e. pathogens

3. Microorganisms that live on dead and decaying organic material are known as

a. germs
b. indigenous microflora
c. parasites
d. pathogens
e. saprophytes

4. The study of algae is called

a. algaeology
b. algonology
c. mycology
d. paleomicrobiology
e. phycology

5. The field of Parasitology involves the study of which of the following types of
organisms?

a. algae
b. arthropods, bacteria, fungi, protozoa, and viruses
c. arthropods, helminths, and certain protozoa
d. bacteria, fungi, and protozoa
e. bacteria, fungi, and viruses

6. Rudolf Virchow is given credit for proposing which of the following theories?

a. abiogenesis
b. biogenesis
c. spontaneous generation
d. the cell theory
e. the germ theory of disease

7. Which of the following microorganisms are considered obligate intracellular pathogens?


a. chlamydias, rickettsias, Mycobacterium leprae, and Treponema pallidum
b. Mycobacterium leprae and Treponema pallidum
c. Mycobacterium tuberculosis and viruses
d. rickettsias, chlamydias, and viruses
e. Treponema pallidum and viruses

8. Which one of the following statements is true?

a. Koch developed a rabies vaccine.


b. Microorganisms are ubiquitous.
c Most microorganisms are harmful to humans.
d. Pasteur conducted experiments that proved the theory of abiogenesis.
e. Pasteurization kills all microorganisms present in milk.

9. Which of the following are even smaller than viruses?

a. chlamydias
b. prions
c. rickettsias
d. viroids
e. both (b) and (d)

10. Which of the following individuals introduced the terms “fermentation,” “aerobes,” and
“anaerobes”?

a. Anton van Leeuwenhoek


b. John Tyndall
c. Louis Pasteur
d. Robert Koch
e. Rudolf Virchow

Chapter 1 Matching Questions from Student Web Site


A. Anton van Leeuwenhoek
B. Robert Koch
C. Louis Pasteur
D. Rudolf Virchow
E. Alexandre Emil Jean Yersin

C 11. Developed vaccines for anthrax and rabies.

D 12. Proposed the theory of biogenesis.

E 13. Discovered the etiologic agent of plague.

A 14. The first person to observe live bacteria and protozoa.

B 15. Developed an experimental procedure that could be used to prove that a specific
microorganism is the cause of a specific infectious disease.

*****

A. pathogens
B. nonpathogens
C. opportunistic pathogens
D. indigenous microflora
E. saprophytes
E 16. Organisms that live on dead and/or decaying organic matter.

B 17. Microorganisms that do not cause disease.

C 18. Microorganisms that usually do not cause disease, but can cause disease under
certain circumstances.

D 19. The microorganisms that live on us and in us.

A 20. The most common causes of infectious diseases or microbial intoxications.

Chapter 1 True/False Questions from Student Web Site


True 21. All infectious diseases are caused by pathogens.

False 22. Pathogens greatly outnumber nonpathogens.

True 23. Using microorganisms to clean up the environment is known as bioremediation.

True 24. Microorganisms are essential in the field of genetic engineering.


False 25. Microorganisms probably appeared on earth about 3.5 million years ago.

False 26. Anton van Leeuwenhoek’s experiments helped to prove that microorganisms
cause disease.

True 27. Louis Pasteur and Robert Koch made significant contributions to the “Germ
Theory of Disease.”

False 28. Pasteurization is a process that kills all microorganisms present in the liquid being
pasteurized.

True 29. Microorganisms contribute more oxygen to our atmosphere than plants do.

True 30. Infectious diseases that are transmitted from animals to humans are known as
zoonoses.

Additional Chapter 1 Matching Questions


A. Robert Koch
B. Louis Pasteur
C. Rudolf Virchow
D. Antony van Leeuwenhoek

C 31. The German scientist who proposed the theory of biogenesis - that life can arise
only from preexisting life

D 32. This Dutchman built single lens microscopes as a hobby and is believed to be the
first person to see live bacteria and protozoa

A 33. This German physician developed a step-by-step scientific method to prove that a
particular microorganism causes a particular disease and contributed to what is
known as "the germ theory of disease"

B 34. This French scientist discovered that microorganisms are responsible for
fermentations, contributed to disproving abiogenesis, and contributed to what is
known as "the germ theory of disease"

CHAPTER 2 QUESTIONS
Chapter 2 Multiple-Choice Questions from Textbook
1. A millimeter is equivalent to __________ nanometers.

a. 100
b. 1,000
c. 10,000
d. 100,000
e. 1,000,000

2. Assume that a pinhead is 1 mm in diameter. How many spherical bacteria (cocci), lined
up side-by-side, would fit across the pinhead. (Hint: Use information from Table 2 - 1.)

a. 10
b. 100
c. 1,000
d. 10,000
e. 100,000

3. What is the length of an average rod-shaped bacterium (bacillus)?

a. 3 mm
b. 3 µm
c. 3 nm
d. 0.3 mm
e. 0.03 mm

4. What is the total magnification when using the high power (high-dry) objective of a
compound light microscope equipped with a 10 ocular lens?

a. 10
b. 40
c. 50
d. 100
e. 400

5. How many times better is the resolution of the transmission electron microscope than the
resolution of the unaided human eye?

a. 100
b. 1,000
c. 10,000
d. 100,000
e. 1,000,000

6. How many times better is the resolution of the transmission electron microscope than the
resolution of the compound light microscope?

a. 100
b. 1,000
c. 10,000
d. 100,000
e. 1,000,000

7. How many times better is the resolution of the transmission electron microscope than the
resolution of the scanning electron microscope?

a. 100
b. 1,000
c. 10,000
d. 100,000
e. 1,000,000

8. The limiting factor of any compound light microscope (i.e., the thing that limits its
resolution to 0.2 µm) is __________.

a. the company you brought it from


b. the number of condenser lenses it has
c. the number of magnifying lenses it has
d. the number of ocular lenses it has
e. the wavelength of visible light

9. Which one of the following individuals is given credit for developing the first compound
microscope?

a. Anton van Leeuwenhoek


b. Hans Jansen
c. Louis Pasteur
d. Marcello Malpighi
e. Robert Hooke

10. A compound light microscope differs from a simple microscope in that the compound
light microscope contains more than one __________.

a. condenser lens
b. light bulb
c. magnifying lens
d. objective lens
e. ocular lens

Chapter 2 Matching Questions from Student Web Site


A. 10
B. 100
C. 1,000
D. 1,000,000
E. 1,000,000,000

C 11. The number of nanometers in a micrometer.

C 12. The resolving power of the compound light microscope is __________ times
better than the resolving power of the unaided eye.

C 13. The number of micrometers in a millimeter.


B 14. The resolving power of the transmission electron microscope is __________
times better than the resolving power of the scanning electron microscope.

D 15. The resolving power of the transmission electron microscope is __________


times better than the resolving power of the unaided eye.

*****

A. 0.2 nm
B. 20 nm
C. 0.2 µm
D. 1 µm
E. 0.2 mm

D 16. The width of a typical coccus.

E 17. The resolving power of the unaided eye.

B 18. The resolving power of the scanning electron microscope.

A 19. The resolving power of the transmission electron microscope.

C 20. The resolving power of the compound light microscope.

Chapter 2 True/False Questions from Student Web Site


False 21. Anton van Leeuwenhoek is given credit for developing the first compound light
microscope.

True 22. The wavelength of visible light limits the size of objects that can be seen with the
compound light microscope.

False 23. The resolving power of compound light microscopes can be improved by adding
additional magnifying lenses.

True 24. A brightfield microscope can be converted to a darkfield microscope by replacing


the condenser on a brightfield microscope with a darkfield condenser.

False 25. Transmission electron microscopes are used to study surface features.

False 26. Primary syphilis is usually diagnosed in the clinical microbiology laboratory by
the use of a scanning electron microscope.

True 27. A magnifying glass could be considered a simple microscope.

True 28. The total magnification achieved when the oil immersion lens is used is X1,000.

True 29. Fluorescence microscopy is often used in immunology laboratories.

False 30. The resolving power of electron microscopes is much better than that of
compound light microscopes because the wavelength of electrons is much longer
than that of visible light.

Additional Chapter 2 Multiple Choice Questions


31. The approximate resolving power of the unaided human eye is ...

a. 0.2 M
b. 0.2 cm
c. 0.2 mm
d. 0.2 µm
e. 0.2 nm

32. The approximate resolving power of the compound light microscope is ...

a. 0.2 M
b. 0.2 cm
c. 0.2 mm
d. 0.2 µm
e. 0.2 nm

33. The reason that better resolution cannot be achieved with the compound light
microscopes used in the lab (i.e., the limiting factor of the compound light microscope) is

a. The "high-tech" capabilities of the company that we purchased them from


b. The efficiency of the workers who assembled the microscopes
c. The manner in which the objective lenses were ground
d. The fact that the eyepiece (ocular) magnification is only 10
e. The wavelength of visible light

34. The total magnification achieved in the laboratory when using the high power (high-dry)
objective of the compound light microscope is ...

a. 10
b. 40
c. 100
d. 400
e. 1000

35. The total magnification achieved in the laboratory when using the oil immersion
objective of the compound light microscope is ...

a. 10
b. 40
c. 100
d. 400
e. 1000

36. Which one of the following statements is not true?

a. The scanning electron microscope is used to examine the surfaces of various


objects and its resolving power is approximately 20 nm
b. The transmission electron microscope is used to examine very thin sections of
various specimens and its resolving power is approximately 1000 times better
than the resolving power of the unaided human eye
c. 0.5 mm = 500 µm = 500,000 nm
d. Better resolution can be achieved using transmission electron microscopes than
can be achieved using scanning electron microscopes

37. The unit of measure usually used to describe bacterial size is ...
a. centimeters
b. millimeters
c. micrometers
d. nanometers
e. angstroms

CHAPTER 3 QUESTIONS

Chapter 3 Multiple-Choice Questions from Textbook


1. Molecules of extrachromosomal DNA are also known as ____________________.

a. Golgi bodies
b. plasmids
c. plastids
d. lysosomes
e. rough ER

2. A bacterium possessing a tuft of flagella at one end of its cell would be called a
____________________ bacterium.

a. amphitrichous
b. lophotrichous
c. monotrichous
d. peritrichous
e. tufty

3. One way in which an archaean would differ from a bacterium is that the archaean would
possess no ____________________.

a. DNA in its chromosome


b. lipids in its cell membrane
c. peptidoglycan in its cell walls
d. RNA in its ribosomes
e. ribosomes in its cytoplasm

4. Some bacteria stain Gram-positive and others stain Gram negative due to differences in
the structure of their ____________________.

a. capsule
b. cell membrane
c. cell wall
d. cytoplasm
e. ribosomes

5. Of the following, which one is not found in prokaryotic cells?

a. chromosome
b. plasmids
c. cell membrane
d. mitochondria
e. ribosomes

6. The 3-Domain System of classification is based upon differences in which one of the
following molecules?

a. DNA
b. mRNA
c. tRNA
d. rRNA
e. peptidoglycan

7. Which of the following is in the correct sequence?

a. Kingdom, Division, Order, Class, Family, Genus


b. Kingdom, Order, Division, Class, Family, Genus
c. Kingdom, Division, Class, Order, Family, Genus
d. Kingdom, Class, Division, Order, Family, Genus
e. Kingdom, Order, Class, Division, Family, Genus

8. Which one of the following is never found in viruses?

a. capsid
b. envelope
c. capsule
d. RNA
e. DNA

9. The semipermeable structure controlling the transport of materials between the cell and
its external environment is the ____________________.

a. cell wall
b. protoplast
c. cytoplasm
d. cell membrane
e. nuclear membrane

10. In eucaryotic cells, ____________________ are the sites of photosynthesis.

a. ribosomes
b. Golgi bodies
c. plasmids
d. plastids
e. mitochondria

Chapter 3 Matching Questions from Student Web Site


A. plastids
B. mitochondria
C. ribosomes
D. endoplasmic reticulum
E. Golgi complex

A 11. Membrane-bound organelles where photosynthesis occurs.

C 12. The sites of protein synthesis in procaryotic cells.

E 13. Considered a “packaging plant,” where proteins are packaged into membrane-
bound vesicles.
B 14. Membrane-bound organelles where energy is produced by the Krebs cycle and
electron transport chain.

C 15. Found in procaryotic cells as well as eucaryotic cells.

*****

A. pili
B. cilia
C. eucaryotic flagella
D. capsules
E. procaryotic flagella

B 16. Short, hairlike projections used as organelles of locomotion by some eucaryotic


cells.

D 17. Found on some bacteria; they serve an antiphagocytic function.

A 18. Found on some bacteria; they enable the bacteria to adhere to surfaces.

E 19. Composed of a protein called flagellin.

C 20. Long, whiplike structures having an internal organization that is described as a


“9+2" arrangement of microtubules.

Chapter 3 True/False Questions from Student Web Site


False 21. The internal structure of procaryotic flagella is the same as the internal structure
of eucaryotic flagella.

True 22. The internal structure of eucaryotic cilia is the same as the internal structure of
eucaryotic flagella.

False 23. The production of endospores by bacteria is a reproductive mechanism.

True 24. Bacteria never have cilia and eucaryotic cells never have pili.

False 25. The 3-Domain System of classification is based on differences in the structure of
transfer RNA (tRNA) molecules.

False 26. One way that archaeans differ from bacteria is that archaeans possess more
peptidoglycan in their cell walls.

False 27. Chitin is found in the cell walls of algae, but is not found in the cell walls of any
other types of microorganisms.

True 28. Tyndallization is a process that kills spores as well as vegetative cells.

True 29. Procaryotic cells do not contain endoplasmic reticulum, Golgi bodies,
mitochondria, plastids, or membrane-bound vesicles.

True 30. In eucaryotic cells, ribosomal RNA (rRNA) molecules are manufactured in the
nucleolus.
Additional Chapter 3 Matching Questions
A. Cellulose
B. Chitin
C. Peptidoglycan
D. Teichoic acids

C 31. Found in the cell walls of both Gram-positive and Gram-negative bacteria, but
greater quantities are found in the cell walls of Gram-positive bacteria

A 32. Found in the cell walls of algae, but not in the cell walls of other microorganisms

D 33. Found in the cell walls of Gram-positive bacteria, but not in the cell walls of
Gram-negative bacteria

B 34. Found in the cell walls of fungi, but not in the cell walls of other microorganisms

C 35. Found in the cell walls of both Gram-positive and Gram-negative bacteria, but
not found in the cell walls of archaeans

*****

A. Pili
B. Slime layer
C. Procaryotic flagella
D. Capsules
E. Eucaryotic flagella

E 36. Have an internal organization described as a "9 +2" arrangement of microtubules

D 37. A type of glycocalyx having an organized structure and found firmly attached to
bacterial cell walls

A 38. Some play a role in bacterial conjugation ("bacterial mating")

D 39. They often serve an antiphagocytic function

C 40. Terms like monotrichous and peritrichous describe their location

A 41. Some of these thin, hairlike appendages enable bacteria to adhere to surfaces

B 42. Some Pseudomonas species produce this type of glycocalyx, which lacks an
organized structure and is only loosely attached to the bacterial cell wall

D 43. Used to make Hib vaccine

*****

A. Mitochondria
B. Ribosomes
C. Endoplasmic reticulum
D. Cilia
E. Plastids

E 44. Membrane-bound organelles where photosynthesis occurs


B 45. The sites of protein synthesis in procaryotic cells

A 46. Membrane-bound organelles where energy is produced by the Krebs cycle and
electron transport chain.

D 47. Short, hairlike projections that are used as organelles of locomotion by some
eucaryotic cells

B 48. Found in procaryotic cells as well as eucaryotic cells

B 49. The sites of protein synthesis in eucaryotic cells

*****

A. Found in both procaryotes and eucaryotes


B. Found only in procaryotes
C. Found only in eucaryotes
D. Not found in either procaryotes or eucaryotes

C 50. Endoplasmic reticulum

D 51. Capsids

A 52. Chromosomes

C 53. Plastids

C 54. Cilia

C 55. Mitochondria

CHAPTER 4 QUESTIONS
Chapter 4 Multiple-Choice Questions from Textbook
1. Which one of the following steps occurs during the multiplication of animal viruses, but
not during the multiplication of bacteriophages?

a. Attachment
b. Penetration
c. Uncoating
d. Biosynthesis
e. Assembly

2. Which one of the following diseases or groups of diseases is not caused by prions?

a. Certain plant diseases


b. “Mad cow disease”
c. Chronic wasting disease of deer and elk
d. Creutzfeldt-Jacob disease of humans
e. Scrapie of sheep

3. Most procaryotic cells reproduce by ____________________.


a. gamete production
b. budding
c. mitosis
d. binary fission
e. spore formation

4. The group of bacteria that lack rigid cell walls and take on irregular shapes is
____________________.

a. rickettsias
b. chlamydias
c. clostridia
d. mycoplasmas
e. mycobacteria

5. At the end of the Gram staining procedure, Gram-positive bacteria will be


____________________.

a. blue-to-purple
b. pink-to-red
c. yellow
d. green
e. orange

6. Which one of the following statements about rickettsias is false?

a. Diseases caused by rickettsias are arthropod-borne.


b. Rocky Mountain spotted fever is caused by a Rickettsia species.
c. Rickettsia species cause typhus and typhus-like diseases.
d. Rickettsias have leaky membranes.
e. Rickets is caused by a Rickettsia species.

7. Which one of the following statements about chlamydias is false?

a. Chlamydias are “energy parasites.”


b. Certain serotypes of Chlamydia trachomatis cause trachoma.
c. Diseases caused by chlamydias are arthropod-borne.
d. Some Chlamydia species cause eye diseases.
e. Some Chlamydia species cause respiratory diseases.

8. Which one of the following statements about cyanobacteria is false?

a. At one time, cyanobacteria were called blue-green algae.


b. Although cyanobacteria are photosynthetic, they do not produce oxygen as a
result of photosynthesis.
c. Some cyanobacteria are important medically because they produce toxins.
d. Some scientists believe that cyanobacteria existed as long ago as 3.5 billion years.
e. Some cyanobacteria are capable of nitrogen fixation.

9. Which one of the following statements about archaeans is false?

a. Some archaeans produce methane.


b. Some archaeans live in extremely hot environments.
c. The cell walls of archaeans contain a thicker layer of peptidoglycan than the
cell walls of bacteria.
d. Archaeans are more closely related to eucaryotes than they are to bacteria.
e. Both archaeans and bacteria are procaryotic organisms.
10. An organism that does not require oxygen, grows better in the absence of oxygen, but can
survive in atmospheres containing some molecular oxygen is known as a/an

a. capnophile
b. microaerophile
c. aerotolerant anaerobe
d. obligate anaerobe
e. facultative anaerobe

Chapter 4 Matching Questions from Student Web Site


A. diplococci
B. diplobacilli
C. staphylococci
D. streptobacilli
E. streptococci

11. Spherical bacteria arranged in pairs are called diplococci.

12. Rod-shaped bacteria arranged in chains are called streptobacilli.

13. Spherical bacteria arranged in clusters are called staphylococci.

14. Rod-shaped bacteria arranged in pairs are called diplobacilli.

15. Spherical bacteria arranged in chains are called streptococci.

*****

A. chlamydias
B. cyanobacteria
C. mycoplasmas
D. rickettsias
E. spirochetes

16. The bacteria that cause syphilis and Lyme disease are spirochetes.

17. Chlamydias are obligate intracellular pathogens that cause diseases such as trachoma,
inclusion conjunctivitis, and urethritis.

18. Cyanobacteria are photosynthetic.


19. Mycoplasmas have no cell walls.

20. Rickettsias are obligate intracellular pathogens that cause diseases such as typhus and
Rocky Mountain spotted fever.

Chapter 4 True/False Questions from Student Web Site


True 21. All diseases caused by Rickettsia spp. are arthropod-borne.

False 22. Viruses contain both DNA and RNA.

False 23. The cell walls of archaeans contain a thicker layer of peptidoglycan than bacterial
cell walls.

False 24. Upon entering a bacterial cell, all bacteriophages immediately initiate the lytic
cycle.

False 25. Mycoplasmas cannot grow on artificial media.

False 26. Viruses are the smallest infectious agents.

True 27. Rickettsia spp. and Chlamydia spp. cannot be grown on artificial media.

False 28. Prions are infectious RNA molecules.

True 29. HIV is an enveloped, double-stranded RNA virus.

True 30. Organisms in the genus Vibrio are curved bacilli.

Additional Chapter 4 Matching Questions


A. Fastidious bacteria
B. Mycoplasma species
C. Clostridium and Bacillus species
D. Mycobacterium and Nocardia species
E. Vibrio species

B 31. Bacteria without cell walls

D 32. Acid-fast bacteria

E 33. Curved, Gram-negative bacilli

C 34. Sporeforming bacteria

A 35. Bacteria having complex nutritional requirements

*****

A. Viroids
B. Virions
C. Prions
D. Temperate phages
E. Virulent phages

E 36. Viruses that infect bacteria and cause the lytic cycle to occur

B 37. Term meaning complete viral particles

A 38. Are known to cause some plant diseases, but no animal diseases

C 39. Infectious protein molecules

A 40. Infectious molecules of RNA

C 41. Cause diseases such as C-J disease, “mad cow disease,” and “scrapie”
*****

A. Most Streptococcus species


B. Staphylococcus species
C. Neisseria species
D. Streptococcus pneumoniae

B 42. Gram-positive cocci in clusters

D 43. Gram-positive diplococci

C 44. Gram-negative diplococci

A 45. Gram-positive cocci in chains

*****

A. Neisseria gonorrhoeae and Campylobacter spp.


B. Clostridium species
C. E. coli, most Staphylococcus and Streptococcus species
D. Mycobacterium and Nocardia spp.
E. Neisseria gonorrhoeae and Haemophilus influenzae

C 46. Facultative anaerobes

A 47. Microaerophiles

B 48. Some species are aerotolerant anaerobes and some species are obligate anaerobes

E 49. Fastidious bacterial pathogens that will grow on chocolate agar but not on blood
agar

A 50. Capnophiles

D 51. Acid-fast bacteria

CHAPTER 5 QUESTIONS

Chapter 5 Multiple-Choice Questions from Textbook


1. Which of the following statements about algae and fungi is/are true?

a. Algae are photosynthetic, whereas fungi are not.


b. Algal cell walls contain cellulose, whereas fungal cell walls do not.
c. Fungal cell walls contain chitin, whereas algal cell walls do not.
d. Some algae and some fungi can cause microbial intoxications.
e. All of the above statements are true.

2. All of the following are algae except ____________________.

a. diatoms
b. dinoflagellates
c. sporozoa
d. Spirogyra
e. desmids
3. All of the following are fungi except ____________________.

a. mushrooms
b. molds
c. yeasts
d. Penicillium
e. Paramecium

4. A protozoan may possess any of the following except ____________________.

a. pseudopodia
b. flagella
c. cytostome
d. cilia
e. hyphae

5. Which one of the following terms is not associated with fungi?

a. pellicle
b. mycelium
c. hyphae
d. mycoses
e. conidia

6. All of the following terms can be used to describe hyphae except ___________________.

a. septate and aseptate


b. reproductive
c. aerial
d. sexual and asexual
e. vegetative

7. A lichen represents a symbiotic relationship between which of the following pairs?

a. an alga and an ameba


b. an alga and a fungus
c. a fungus and an ameba
d. an alga and a cyanobacterium
e. a yeast and an ameba

8. A stigma is a ____________________.

a. thickened membrane
b. primitive mouth
c. light-sensing organelle
d. type of plastid
e. type of flagellum

9. If a dimorphic fungus is causing a respiratory infection, which of the following might be


seen in a sputum specimen from that patient?

a. yeasts
b. hyphae
c. amebae
d. conidia
e. cysts
10. Which one of the following is not a fungus?
a. Aspergillus
b. Prototheca
c. Penicillium
d. Candida
e. Cryptococcus

Chapter 5 Matching Questions from Student Web Site


A. algae
B. fungi
C. lichens
D. protozoa
E. slime molds

11. Yeasts and molds are examples of fungi.

12. All algae and lichens are photosynthetic.

13. Protozoa are classified by their means of locomotion.

14. The cell walls of fungi contain chitin.

15. The cell walls of algae contain cellulose.

16. Algae rarely cause infectious diseases, but cause a variety of microbial intoxications.

17. Lichens are examples of a symbiotic relationship known as mutualism.

18. Protozoa cause diseases such as African sleeping sickness, babesiosis, cryptosporidiosis,
malaria, and toxoplasmosis.

19. Diseases caused by fungi are called mycoses.

20. Toxins produced by algae are called phycotoxins.

Chapter 5 True/False Questions from Student Web Site


True 21. Slime molds possess characteristics of both fungi and protozoa.

False 22. Protozoa in the category known as Mastigophora move by means of cilia.

False 23. A dimorphic fungus would exist as a mold inside the human body.

True 24. The organism that causes a “red tide” is an alga.

True 25. Volvox is a multicellular alga.

False 26. A stigma is an organelle that pumps water out of the cell.

False 27. Sexual spores are also known as conidia.

False 28. Classification of fungi is based upon the type of conidia that they produce.
True 29. Protozoa that move by means of pseudopodia are in a category known as
Sarcodina.

True 30. Fungi can cause both infectious diseases and microbial intoxications.

Additional Chapter 5 Matching Questions


A. Prototheca
B. Pfiesteria piscicida
C. Dinoflagellates
D. A red marine alga
E. Euglena

B 31. Has caused massive fish kills along the east coast of the United States

C 32. Associated with red tides and paralytic shellfish poisoning

D 33. The source of agar

A 34. The only alga known to cause an infectious disease

E 35. Classified as an alga by some taxonomists and a protozoan by others

CHAPTER 6 QUESTIONS

Chapter 6 Multiple-Choice Questions from Textbook


1. Which of the following are the “building blocks” of proteins?

a. monosaccharides
b. amino acids
c. nucleotides
d. fatty acids
e. peptides

2. Glucose, sucrose and cellulose are examples of ____________________.

a. polypeptides
b. polysaccharides
c. carbohydrates
d. monosaccharides
e. disaccharides

3. Which one of the following nitrogenous bases is not found in an RNA molecule?

a. adenine
b. guanine
c. cytosine
d. thymine
e. uracil

4. Which of the following are purines?


a. adenine and thymine
b. guanine and cytosine
c. cytosine and uracil
d. adenine and guanine
e. thymine and uracil

5. Which one of the following is not found at the site of protein synthesis?

a. DNA
b. mRNA
c. rRNA
d. tRNA
e. a ribosome

6. Which of the following statements about DNA is/are true?

a. Within cells, DNA molecules are usually double-stranded.


b. DNA molecules contain deoxyribose.
c. DNA contains thymine but not uracil.
d In a double-stranded DNA molecule, adenine on one strand will be connected to
thymine on the complimentary strand by a hydrogen bond.
e. All of the above statements are true.

7. The amino acids in a polypeptide chain are connected by ____________________.

a. glycosidic bonds
b. hydrogen bonds
c. peptide bonds
d. covalent bonds
e. both C and D

8. Which of the following statements about nucleotides is/are true?

a. A nucleotide contains a pentose.


b. A nucleotide contains a nitrogenous base.
c. Nucleotides can bind to other nucleotides by covalent bonds or hydrogen bonds.
d. A nucleotide contains a phosphate group.
e. All of the above statements are true.

9. A heptose contains how many carbon atoms?

a. 3
b. 4
c. 5
d. 6
e. 7

10. Virtually all enzymes are ____________________.

a. carbohydrates
b. nucleic acids
c. substrates
d. proteins
e. lipids
Chapter 6 Matching Questions from Student Web Site
A. amino acids
B. disaccharides
C. fatty acids
D. monosaccharides
E. nucleotides

11. Amino acids are the “building blocks” of proteins.

12. Nucleotides consist of a nitrogenous base, a pentose, and a phosphate group.

13. Monosaccharides are the “building blocks” of polysaccharides.

14. Sucrose, lactose, and maltose are examples of disaccharides.

15. Fructose, galactose, and glucose are examples of monosaccharides.

*****
A. a dehydration synthesis reaction
B. DNA replication
C. a hydrolysis reaction
D. transcription
E. translation

16. The end product of transcription is a messenger RNA molecule.

17. Combining two monosaccharides to form a disaccharide is an example of a dehydration


synthesis reaction.

18. James Watson and Francis Crick were the first to publish an article describing DNA
replication.

19. The end product of translation is a protein.

20. Breaking the bond in a disaccharide to produce two monosaccharides is an example of a


hydrolysis reaction.

Chapter 6 True/False Questions from Student Web Site


True 21. The covalent bonds that hold monosaccharides together in a polysaccharide are
called glycosidic bonds.

False 22. A DNA nucleotide consists of the following three parts: a nitrogenous base,
ribose, and a phosphate group.

True 23. The waxes in the cell walls of Mycobacterium tuberculosis cause this organism to
be acid-fast.

True 24. The basic structure of a cell membrane is a lipid bilayer.

False 25. DNA polymerase is the only enzyme required for DNA replication.

False 26. Genes that are expressed only when needed are called constitutive genes.
True 27. Polysaccharides, polypeptides, and nucleic acids are all examples of polymers.

False 28. During translation, amino acids are “activated” by attaching to an appropriate
rRNA molecule.

True 29. The peptide bonds that hold amino acids together in protein molecules are
examples of covalent bonds.

True 30. In double-stranded DNA molecules, the two strands are held together by
hydrogen bonds.

Additional Chapter 6 Matching Questions


A. DNA polymerase
B. RNA polymerase
C. mRNA
D. rRNA
E. tRNA

A 31. A protein that plays a role in DNA replication

C 32. The end product of transcription

B 33. A protein that plays a role in transcription

C 34. The location of codons

E 35. The location of anticodons

D 36. An integral part of ribosomes, whether or not they are participating in protein
synthesis

D 37. Produced in the nucleolus of eucaryotic cells

E 38. The molecule responsible for "activating" amino acids, thus enabling them to be
added to the growing polypeptide chain

*****

A. Polypeptides
B. Polysaccharides
C. Nucleic acids
D. Substrates

B 39. The “building blocks" are held together by glycosidic bonds

C 40. Composed of “building blocks” that contain phosphorous

A 41. Polymers that are composed of amino acids

A 42. Enzymes are examples

B 43. Cellulose, glycogen and starch are examples

A 44. The “building blocks" are held together by peptide bonds


*****

A. Sucrose
B. Lactose
C. Glucose
D. Ribose
E. Maltose

B 45. The disaccharide found in MacConkey agar

D 46. A pentose

A 47. A disaccharide composed of glucose plus fructose

C 48. Another name for dextrose

C 49. A hexose

Additional Chapter 6 Questions


Assume that you are taking an Organic Chemistry class. Your teacher has given you four organic
compounds to analyze. She tells you that one is a carbohydrate, one is a hydrocarbon, one is a
nucleic acid, and one is a protein.

50. Compound A contains only carbon and hydrogen. Of the four types of compounds
which were given to you to analyze, which one best describes Compound A?

Compound A is a hydrocarbon

51. You discover that Compound B contains only carbon, hydrogen, and oxygen. Of
the four types of compounds which were given to you to analyze, which one best
describes Compound B?

Compound B is a carbohydrate_

52. You discover that Compound C contains only carbon, hydrogen, oxygen, and
nitrogen. Of the four types of compounds which were given to you to analyze,
which one best describes Compound C?

Compound C is a protein

53. You discover that Compound D contains only carbon, hydrogen, oxygen,
nitrogen, and phosphorous. Of the four types of compounds which were given to
you to analyze, which one best describes Compound D?

Compound D is a nucleic acid

CHAPTER 7 QUESTIONS
Chapter 7 Multiple-Choice Questions from Textbook
1. Which one of the following characteristics do animals, fungi, and protozoa have in
common?
a. They obtain their carbon from light
b. They obtain their energy and carbon atoms from chemicals
c. They obtain their carbon from carbon dioxide
d. They obtain their energy from light
e. They obtain their carbon from inorganic compounds

2. The largest number of ATP molecules are produced during which phase of aerobic
respiration?

a. the Krebs cycle


b. the electron transport chain
c. glycolysis
d. the citric acid cycle
e. fermentation

3. Which one of the following processes does not involve bacteriophages?

a. lysogeny
b. lysogenic conversion
c. transduction
d. transformation
e. the lytic cycle

4. In transduction, bacteria acquire new genetic information in the form of


____________________.

a. bacterial genes
b. viral genes
c. “naked DNA”
d. mutations
e. R-factors

5. The process whereby "naked DNA” is absorbed into a bacterial cell is known as
___________________.

a. transplantation
b. transformation
c. transduction
d. transcription
e. translation

6. In lysogenic conversion, bacteria acquire new genetic information in the form of


____________________.

a. bacterial genes
b. viral genes
c. “naked DNA”
d. mutations
e. R-factors

7. Saprophytic fungi are able to digest organic molecules outside of the organism by means
of ____________________.

a. coenzymes
b. endoenzymes
c. holoenzymes
d. exoenzymes
e. apoenzymes

8. The process by which a nontoxigenic Corynebacterium diphtheriae cell is changed into a


toxigenic cell is called ____________________.

a. conjugation
b. transformation
c. prestidigitation
d. lysogenic conversion
e. transduction

9. Which of the following does/do not occur in anaerobes?

a. glycolysis
b. fermentation reactions
c. catabolic reactions
d. electron transport system
e. anabolic reactions

10. Proteins that must link up with a cofactor in order to function as an enzyme are called
____________________.

a. coenzymes
b. endoenzymes
c. holoenzymes
d. exoenzymes
e. apoenzymes

Chapter 7 Matching Questions from Student Web Site


A. autotrophs
B. heterotrophs
C. lithotrophs
D. organotrophs
E. phototrophs

11. Lithotrophs are chemotrophs that use inorganic chemicals as their energy source.

12. Organisms that use organic compounds as their source of carbon are called heterotrophs.

13. Organisms that use organic compounds as their energy source are called organotrophs.

14. Organisms that use carbon dioxide as their source of carbon are called autotrophs.

15. Organisms that use light as their energy source are called phototrophs.

*****

A. conjugation
B. lysogenic conversion
C. mutation
D. transduction
E. transformation

16. In lysogenic conversion, bacteria acquire new genetic information in the form of viral
genes.
17. Oswald Avery and his colleagues discovered that DNA is the hereditary molecule while
performing transformation experiments with Streptococcus pneumoniae.

18. In transformation, bacteria acquire new genetic information as a result of absorbing


pieces of “naked DNA” from their environment.

19. In conjugation, genetic information is passed from one bacterial cell to another via a
hollow sex pilus.

20. In transduction, bacteria acquire new genetic information when bacteriophages inject
bacterial genes.

Chapter 7 True/False Questions from Student Web Site


True 21. Dehydration synthesis reactions always involve the removal of a molecule of
water.

False 22. The biosynthesis of polysaccharides, polypeptides, and nucleic acids are examples
of catabolic reactions.

True 23. Oxidation-reduction reactions are paired reactions that involve the transfer of
electrons.

True 24. Breaking a disaccharide down into its two monosaccharide components is an
example of a hydrolysis reaction.

False 25. Anabolic reactions are a cell’s major source of energy.

False 26. The majority of the energy produced in aerobic respiration is produced by the
Krebs cycle.

True 27. In glycolysis, a 6-carbon glucose molecule is broken down into two 3-carbon
molecules of pyruvic acid.

True 28. Aerobic respiration is a more efficient method of breaking down glucose than is
fermentation.

False 29. Virulent bacteriophages are responsible for lysogenic conversion.

False 30. Mutations are always harmful.

Additional Chapter 7 Matching Questions


A. Conjugation
B. Transduction
C. Transformation
D. Transcription
E. Lysogenic conversion

B 31. Transfer of bacterial genes from one bacterial cell to another by a bacteriophage

A 32. Transfer of genetic material from a donor cell to a recipient cell through a sex
pilus
E 33. The process by which a non-toxigenic Corynebacterium diphtheriae cell becomes
a toxigenic cell

C 34. Occurs only in bacterial cells described as being "competent"

C 35. Acquisition of new genetic material by the uptake of "naked" DNA

*****

A. Catabolic reactions
B. Anabolic reactions
C. Oxidation-reduction reactions
D. Aerobic respiration
E. Fermentation

C 36. Paired reactions that involve the transfer of electrons

E 37. Biochemical pathway involving glycolysis and the production of ethyl alcohol,
lactic acid, or other products from pyruvic acid; no Krebs cycle or electron
transport chain involved; does not require oxygen

B 38. Used to build complex organic compounds from simpler compounds; also known
as biosynthetic reactions; chemical bonds are created; energy is required

D 39. Biochemical pathway involving glycolysis, the Krebs cycle, and the electron
transport chain

A 40. Used to break complex organic compounds into simpler compounds; also known
as degradative reactions; chemical bonds are broken; energy is released

D 41. Which produces more energy: "D" or "E"?

D 42. Cannot take place in an obligate anaerobe

CHAPTER 8 QUESTIONS
Chapter 8 Multiple-Choice Questions from Textbook
1. It would be necessary to use a tuberculocidal agent to kill a particular species of
____________________.

a. Pseudomonas
b. Clostridium
c. Staphylococcus
d. Mycobacterium
e. Streptococcus

2. Pasteurization is a type of ____________________ technique.

a. surgical aseptic
b. disinfection
c. sterilization
d. antiseptic
e. medical aseptic

3. The combination of freezing and drying is known as ____________________.

a. desiccation
b. tyndallization
c. pasteurization
d. sterilization
e. lyophilization

4. Organisms that live in and around hydrothermal vents at the bottom of the ocean are
____________________.

a. halophilic, alkaliphilic, and psychrophilic


b. halophilic, thermophilic, and barophilic
c. acidophilic, psychrophilic, and halophilic
d. halophilic, psychrophilic, and barophilic
e. mesophilic, halophilic, and barophilic

5. When placed into a hypertonic solution, a bacterial cell will ____________________.

a. swell
b. shrink
c. lyse
d. hemolyze
e. become crenated

6. To prevent Clostridium infections in a hospital setting, a ____________________


disinfectant should be used.

a. virucidal
b. fungicidal
c. sporicidal
d. tuberculocidal
e. pseudomonicidal

7. Sterilization can be accomplished by use of ____________________.

a. antiseptics
b. disinfectants
c. an autoclave
d. medical aseptic techniques
e. pasteurization

8. The goal of medical asepsis is to kill ____________________, whereas the goal of


surgical asepsis is to kill ____________________.

a. all microorganisms ..... pathogens


b. nonpathogens ..... pathogens
c. bacteria ..... bacteria and viruses
d. pathogens ..... all microorganisms
e. pathogens ..... nonpathogens

9. Which one of the following types of culture media is selective and differential?

a. Blood agar
b. Thayer-Martin agar
c. Phenylethyl alcohol agar
d. MacConkey agar
e. Sabouraud agar

10. All of the following types of culture media are enriched and selective except
____________________.

a. Blood agar
b. Thayer-Martin agar
c. Phenylethyl alcohol agar
d. Colistin-nalidixic acid agar
e. New York City agar

Chapter 8 Matching Questions from Student Web Site


A. capnophiles
B. fastidious organisms
C. halophiles
D. microaerophiles
E. psychrotrophs

11. Organisms that prefer salty environments are called halophiles.

12. Organisms that have especially demanding nutritional requirements are called fastidious
organisms.

13. Organisms that prefer environments having decreased concentrations of oxygen are
called microaerophiles.

14. Organisms that prefer refrigerator temperature are called psychrotrophs.

15. Organisms that grow best in environments containing increased concentrations of carbon
dioxide are called capnophiles.

*****

A. Blood agar
B. Chocolate agar
C. MacConkey agar
D. Phenylethyl alcohol agar
E. Thayer-Martin agar

16. Although Neisseria gonorrhoeae and Haemophilus influenzae will not grow on blood
agar, they will grow on chocolate agar.

17. Thayer-Martin agar is a highly enriched and highly selective medium, used to isolate
Neisseria gonorrhoeae from clinical specimens.

18. Blood agar is an example of a medium that is both enriched and differential.

19. Phenylethyl alcohol agar is selective for Gram-positive bacteria.

20. MacConkey agar is an example of a medium that is selective and differential.


Chapter 8 True/False Questions from Student Web Site
False 21. A bacterial cell would swell and burst if placed in an extremely hypertonic
solution.

False 22. Pasteurization is a method of sterilizing liquids.

True 23. Archaeans that live in or near hydrothermal vents at the bottom of the ocean are
halophilic, barophilic, and thermophilic.

False 24. Lyophilization is an excellent method of killing microorganisms.

True 25. Antiseptic technique is a type of aseptic technique.

False 26. In an autoclave, microorganisms are killed by ethylene oxide.

False 27. The goal of disinfection is to kill all microorganisms.

False 28. It is not possible to culture protozoa in the laboratory.

False 29. Rapid freezing, using liquid nitrogen, is an excellent method of killing
microorganisms.

True 30. Microorganisms can be killed by direct exposure to ultraviolet light.

Additional Chapter 8 Matching Questions


A. Enriched and differential
B. Enriched and selective
C. Selective and differential
D. Selective only
E. Differential only

B 31. Phenylethyl alcohol agar and Thayer-Martin agar

C 32. MacConkey agar and mannitol salt agar

D 33. Sabouraud dextrose agar

A 34. Blood agar

*****

A Disinfection
B. Pasteurization
C. Sanitization
D. Sterilization

D 35. The complete destruction of all living organisms, including cells, spores, and
viruses

A 36. The use of antiseptics is an example of a __________ technique

D 37. Autoclaving for 15 to 20 minutes at 15 psi and 121oC is one method of


accomplishing this
A 38. The destruction or removal of pathogens by physical or chemical methods

B 39. Heating to 60oC for 30 minutes is one method of accomplishing this; sufficient to
kill pathogens and reduce total bacterial counts

D 40. Heating in a 160oC oven for 2 hours would accomplish this

C 41. The reduction of microbial populations to levels considered safe by public health
standards

*****

A. Mesophiles
B. Thermophiles
C. Psychrophiles
D. Psychrotrophs
E. Halophiles

B 42. The brightly colored algae that grow in the hot springs at Yellowstone National
Park are examples

E 43. Organisms in this category could grow on mannitol-salt-agar

A 44. Most human pathogens are in this category

E 45. Bacteria that live in the ocean are examples, regardless of the temperature of the
water

D 46. A specific group of microorganisms whose optimal growth temperature is 4oC

E 47. Vibrio cholerae is an example

*****

A. Hypertonic
B. Hypotonic
C. Isotonic
D. Gin and tonic

A 48. Crenation of red blood cells would occur if they were placed into a __________
solution

A 49. Adding large quantities of salt or sugar to various types of foods is a way of
preserving them. If bacteria should enter such foods, they would find themselves
in a __________ environment

B 50. If red blood cells were placed into a __________ solution, they might hemolyze

CHAPTER 9 QUESTIONS
Chapter 9 Multiple-Choice Questions from Textbook
1. Which one of the following is least likely to be taken into consideration when deciding
which antibiotic to prescribe for a patient?
a. the patient’s age
b. the patient’s weight
c. pregnancy
d. the patient’s underlying medical conditions
e. other medications that the patient is taking

2. Which one of the following is least likely to lead to drug resistance in bacteria?

a. a chromosomal mutation that alters the shape of a particular drug binding site
b. a chromosomal mutation that alters cell membrane permeability
c. receiving a gene that codes for an enzyme that destroys a particular antibiotic
d. receiving a gene (or genes) that codes for a MDR pump
e. receiving a gene that codes for the production of a capsule

3. Which one of the following is not a common mechanism by which antimicrobial agents
kill or inhibit the growth of bacteria?

a. inhibition of cell wall synthesis


b. damage to cell membranes
c. inhibition of nucleic acid synthesis (either DNA or RNA)
d. inhibition of protein synthesis
e. destruction of capsules

4. Multi-drug therapy is always employed when a patient is diagnosed as having


____________________.

a. tuberculosis
b. an infection caused by MRSA
c. syphilis
d. strep throat
e. diphtheria

5. Which one of the following terms or names has nothing to do with the use of two drugs
simultaneously?

a. Septra®
b. synergism
c. Bactrim®
d. antagonism
e. Salvarsan®

6. Which one of the following is not a common mechanism by which antifungal agents
work?

a. by dissolving hyphae
b. by binding with cell membrane sterols
c. by interfering with sterol synthesis
d. by blocking mitosis
e. by blocking nucleic acid synthesis

7. Which one of the following scientists discovered penicillin?

a. Paul Ehrlich
b. Sir Howard Walter Florey
c. Alexander Fleming
d. Gerhard Domagk
e. Selman Waksman

8. Which one of the following scientists is considered to be the “Father of Chemotherapy”?

a. Paul Ehrlich
b. Sir Howard Walter Florey
c. Alexander Fleming
d. Gerhard Domagk
e. Selman Waksman

9. All of the following antimicrobial agents work by inhibiting cell wall synthesis except
____________________.

a. penicillin
b. bacitracin
c. cephalosporins
d. chloramphenicol
e. vancomycin

10. All of the following antimicrobial agents work by inhibiting protein synthesis except
____________________.

a. tetracycline
b. erythromycin
c. chloramphenicol
d. streptomycin
e. imipenem

Chapter 9 Matching Questions from Student Web Site


A. Bactrim® and Septra®
B. chloramphenicol, erythromycin, and tetracycline
C. isoniazid, rifampin, pyrazinamide, ethambutol, and streptomycin
D. methicillin, nafcillin, oxacillin, and cloxacillin
E. penicillin and cephalosporin

11. Penicillin and cephalosporin are examples of antibiotics that are produced by fungi.

12. Methicillin, nafcillin, oxacillin, and cloxacillin are in a class of drugs known as
penicillinase-resistant penicillins.

13. Isoniazid, rifampin, pyrazinamide, ethambutol, and streptomycin are examples of


drugs that are used to treat tuberculosis.

14. Chloramphenicol, erythromycin, and tetracycline are examples of drugs that inhibit
protein synthesis.

15. Penicillin and cephalosporin are examples of drugs that could be destroyed by beta-
lactamases.

*****

A. a chemotherapeutic agent
B. an antibiotic
C. an antifungal agent
D. an antiviral agent
E. an antimicrobial agent

16. Acyclovir is specifically used as an antiviral agent.

17. A chemotherapeutic agent is any chemical or drug that is used to treat any disease or
medical condition.

18. An antibiotic is a substance produced by a microorganism that is effective in killing or


inhibiting the growth of another species of microorganism.

19. Any drug used to treat an infectious disease is called an antimicrobial agent.

20. Amphotericin B is specifically used as an antifungal agent.

Chapter 9 True/False Questions from Student Web Site


False 21. Strains of Staphylococcus aureus known as MRSA are resistant to methicillin, but
are susceptible to most other antibacterial agents.

True 22. Bacteria can develop resistance to a particular antimicrobial agent as a result of a
chromosomal mutation or the acquisition of a new gene.

True 23. A bacterial cell that receives an R-factor becomes multi-drug resistant.

False 24. A “superinfection” is an infection that cannot be cured.

False 25. Using two different antimicrobial agents to treat a patient’s infection is referred to
as antagonism if the result that is achieved is much better than that which could
have been achieved using only one of the drugs.

False 26. Because he discovered penicillin, Alexander Fleming is often referred to as the
“Father of Chemotherapy.”

True 27. Bacteria can become drug resistant as a result of transduction, transformation, or
conjugation.

False 28. All antimicrobial agents are antibiotics.

True 29. Administering an antibiotic to a patient to treat one type of infectious disease
could actually cause other types of infectious diseases in that patient.

True 30. Bacteria that acquire the genes that code for an MDR pump become multi-drug
resistant.

Additional Chapter 9 Matching Questions


A. Penicillin, bacitracin, cephalosporins, vancomycin
B. Ampicillin and carbenicillin
C. Methicillin, oxacillin, cloxacillin, nafcillin
D. Tetracycline, chloramphenicol, erythromycin, streptomycin
E. Penicillin and cephalosporins, but not bacitracin and vancomycin

C 31. Semisynthetic penicillins that are known as penicillinase-resistant penicillins


A 32. Examples of antibiotics that inhibit cell wall synthesis

B 33. Semisynthetic penicillins that are not resistant to penicillinase

D 34. Examples of antibiotics that inhibit protein synthesis

E 35. Examples of antibiotics produced by molds

*****
A. Enterococcus species
B. MRSA
C. Clostridium difficile
D. Escherichia coli
E. Enterobacter species

C 36. A superinfection with this organism can lead to antibiotic-associated diarrhea and
pseudomembranous colitis

B 37. The only drugs that can be used to treat infections caused by these Gram-positive
cocci in clusters are vancomycin, Synercid® and Zyvox®; unfortunately, some
strains have even developed resistance to these antibiotics

A 38. “Superbugs” known as VRE are vancomycin-resistant strains of __________.

CHAPTER 10 QUESTIONS

Chapter 10 Multiple-Choice Questions from Textbook


1. A symbiont could be a/an ____________________.

a. commensal
b. opportunist
c. parasite
d. endosymbiont
e. all of the above

2. The greatest number and variety of indigenous microflora of the human body live in or
on the ____________________.

a. skin
b. nasal passages
c. mouth
d. colon
e. genitourinary tract

3. Escherichia coli living in the human colon can be considered to be a/an


___________________.

a. opportunist
b. endosymbiont
c. enteric bacillus
d. symbiont in a mutualistic relationship
e. all of the above

4. Which one of the following sites of the human body does not have indigenous
microflora?

a. the distal urethra


b. the vagina
c. the colon
d. the bloodstream
e. the skin

5. Which of the following would be present in highest numbers in the indigenous microflora
of the human mouth?

a. beta-hemolytic streptococci
b. alpha-hemolytic streptococci
c. Staphylococcus aureus
d. Staphylococcus epidermidis
e. Candida albicans

6. Which of the following would be present in highest numbers in the indigenous microflora
of the skin?

a. Pseudomonas aeruginosa
b. Escherichia coli
c. Enterococcus spp.
d. coagulase-negative staphylococci
e. Candida albicans

7. Which of the following are least likely to play a role in the nitrogen cycle?

a. saprophytes
b. enteric bacilli
c. organisms living in the root nodules of legumes
d. nitrifying and denitrifying bacteria
e. nitrogen-fixing bacteria

8. Microorganisms are used in which of the following industries?

a. antibiotic industry
b. food industry
c. wine industry
d. chemical industry
e. all of the above

9. The term that best describes a symbiotic relationship in which two different
microorganisms occupy the same ecological niche, but have absolutely no effect upon
each other is ____________________.

a. commensalism
b. biofilm
c. parasitism
d. mutualism
e. neutralism

10. All of the following are members of the family Enterobacteriaceae except
____________________.
a. Escherichia spp.
b. Enterobacter spp.
c. Enterococcus spp.
d. Proteus spp.
e. Klebsiella spp.

Chapter 10 Matching Questions from Student Web Site


A. commensalism
B. mutualism
C. neutralism
D. parasitism
E. synergism

11. When two microorganisms occupying the same environmental niche have absolutely no
effect upon each other, it is known as neutralism.

12. Bacterial vaginosis is an example of synergism.

13. Commensalism is a symbiotic relationship of benefit to one of the symbionts, but neither
beneficial nor harmful to the other.

14. Parasitism is a symbiotic relationship of benefit to one of the symbionts, and detrimental
to the other.

15. A lichen is a classic example of mutualism.

*****

A. cyanobacteria
B. denitrifying bacteria
C. nitrifying bacteria
D. nitrogen-fixing bacteria
E. saprophytes

16. In the nitrogen cycle, bacteria called nitrogen-fixing bacteria convert atmospheric
nitrogen gas into ammonia in the soil.

17. Nitrogen-fixing bacteria live in the root nodules of legumes such as alfalfa and clover.

18. Saprophytes are capable of converting the nitrogen within a dead plant or animal into
ammonia in the soil.

19. In the nitrogen cycle, soil organisms called nitrifying bacteria convert ammonia into
nitrites, and nitrites into nitrates.

20. In the nitrogen cycle, soil organisms called denitrifying bacteria convert the nitrogen in
nitrates to nitrogen gas in the atmosphere.

Chapter 10 True/False Questions from Student Web Site


False 21. No microorganisms are able to live in the stomach, due to the extremely low pH
of the stomach contents.
False 22. Microbial communities known as biofilms are interesting, but they have no
medical significance.

False 23. Microorganisms are unable to live in the colon due to the lack of oxygen there.

True 24. Some of the bacteria used in bioremediation are naturally-occurring, but others
have been genetically engineered.

True 25. Many of the members of our indigenous microflora have the potential to cause
disease.

True 26. There could be as many as 100 trillion microorganisms that live on us and in us.

False 27. The most common organisms in the indigenous microflora of the mouth are
various species of beta-hemolytic streptococci.

True 28. Microbes cause thousands of different types of plant diseases.

True 29. Most relationships between humans and microbes are beneficial rather than
harmful.

True 30. Beneficial microorganisms far outnumber harmful ones.

Additional Chapter 10 Matching Questions


A. Commensalistic
B. Mutualistic
C. Parasitic
D. Saprophytic
E. Synergistic

D 31. Organisms that live on dead and decaying organic matter are said to be
__________

B 32. The type of relationship that termites have with the cellulose-eating protozoa that
live in their intestinal tract

B 33. A symbiotic relationship that is beneficial to both parties (symbionts) is said to be


a __________ relationship

B 34. A lichen is an example of a __________ relationship

A 35. A symbiotic relationship that is beneficial to one party (symbiont), but of no


consequence to the other is said to be a __________ relationship

B 36. The type of relationship that humans have with the Escherichia coli organisms
that live in their colon

C 37. A symbiotic relationship that is beneficial to one party (symbiont), and


detrimental to the other is said to be a __________ relationship

*****

A. Alpha-hemolytic streptococci
B. Coagulase-negative staphylococci
C. Lactobacillus spp.
D. Escherichia coli
E. Candida albicans

C 38. The most common member or members of the indigenous microflora of the
vagina

B 39. The most common member or members of the indigenous microflora of the skin

A 40. The most common member or members of the indigenous microflora of the
mouth

*****

A. Decomposers and saprophytes


B. Denitrifying bacteria
C. Nitrifying bacteria
D. Nitrogen-fixing bacteria
E. Nitrogen-unfixing bacteria

D 41. Some of these bacteria live in and near root nodules of plants called legumes,
including alfalfa, clover, peas, soy beans and peanuts

D 42. Convert nitrogen in the air into ammonia in the soil by a process called nitrogen-
fixation

A 43. Convert the nitrogen that is found in dead plants and animals to ammonia in the
soil by a process called ammonification

B 44. Convert nitrates in the soil to nitrogen in the air by a process called denitrification

C 45. Convert ammonia in the soil to nitrites and nitrates by a process called
nitrification

CHAPTER 11 QUESTIONS

Chapter 11 Multiple-Choice Questions from Textbook


1. Which of the following terms best describes chlamydial genital infection in the United
States?

a. a sporadic disease
b. a nonendemic disease
c. a contagious disease
d. an epidemic disease
e. a pandemic disease

2. Which of the following are considered reservoirs of infection?

a. contaminated food
b. contaminated drinking water
c. carriers
d. rabid animals
e. all of the above
3. The most common nationally notifiable infectious disease in the United States is
____________________.

a. gonorrhea
b. the common cold
c. syphilis
d. chlamydial genital infections
e. tuberculosis

4. Which of the following arthropods is the vector of Lyme disease?

a. mosquito
b. head louse
c. tick
d. flea
e. mite

5. The most common zoonotic disease in the United States is ____________________.

a. plague
b. Rocky Mountain spotted fever
c. Lyme disease
d. salmonellosis
e. rabies

6. Which one of the following organisms in not one of the four most likely potential
biological warfare or bioterrorist agents?

a. Ebola virus
b. Yersinia pestis
c. Bacillus anthracis
d. Variola major
e. Clostridium botulinum

7. All of the following are major steps in the treatment of a community’s drinking water
except ____________________.

a. filtration
b. sedimentation
c. flocculation
d. boiling
e. chlorination

8. The largest waterborne epidemic ever to occur in the United States occurred in which one
of the following cities?

a. New York City


b. Atlanta
c. Chicago
d. Los Angeles
e. Milwaukee

9. Typhoid fever is caused by a species of ____________________.

a. Escherichia
b. Salmonella
c. Shigella
d. Campylobacter
e. Vibrio

10. Which of the following associations is incorrect?

a. malaria ... mosquito


b. plague ... flea
c. Rocky Mountain spotted fever ... mite
d. viral encephalitis ... mosquito
e. ehrlichiosis ... tick

Chapter 11 Matching Questions from Student Web Site


A. communicable diseases
B. endemic diseases
C. epidemic diseases
D. pandemic diseases
E. sporadic diseases

11. Diseases that are always present in a population are known as endemic diseases.

12. Diseases that are transmissible from person to person are known as communicable
diseases.

13. Diseases that occur only occasionally in a particular population are known as sporadic
diseases.

14. Because large numbers of cases of AIDS, malaria, and tuberculosis are presently
occurring in many different countries, they are known as pandemic diseases.

15. Diseases with unusually high numbers of cases that often occur in one particular
geographic location are known as epidemic diseases.

*****

A. biting flies
B. bugs
C. lice
D. mosquitos
E. ticks

16. The etiologic agents of dengue fever, filariasis, malaria, West Nile encephalitis, and
yellow fever are all transmitted by mosquitos.

17. Ticks transmit the etiologic agents of babesiosis, ehrlichiosis, Lyme disease, relapsing
fever, and Rocky Mountain spotted fever.

18. The etiologic agent of American trypanosomiasis (Chagas’ disease) is transmitted by


arthropods in a class of insects known as bugs.

19. Lice transmit the etiologic agents of epidemic typhus and trench fever.

20. African sleeping sickness, leishmaniasis, and onchocerciasis are transmitted by various
types of biting flies.
Chapter 11 True/False Questions from Student Web Site
True 21. Influenza is an example of a contagious disease.

False 22. Zoonotic diseases are diseases that humans acquire from zoo animals.

False 23. The largest waterborne outbreak ever to occur in the United States was caused by
Giardia lamblia.

True 24. Water containing 1 coliform per 100 mL would be considered potable.

False 25. The most common zoonotic infection in the United States is Rocky Mountain
spotted fever.

True 26. Soil can contain the spores that cause botulism, gas gangrene, and tetanus.

True 27. Chlamydial genital infections and gonorrhea are the two most common nationally
notifiable infectious diseases in the United States.

False 28. The levels of chlorine routinely used for water treatment are sufficient to kill
Giardia cysts and Cryptosporidium oocysts.

True 29. Yersinia pestis, the bacterium that causes plague, is one of the pathogens most
often discussed as a potential biological weapon.

True 30. Gonorrhea is considered to be a communicable disease, but not a contagious


disease.

Additional Chapter 11 Matching Questions


A. Endemic disease
B. Epidemic disease
C. Pandemic disease
D. Sporadic disease

B 31. Several dozen people having nausea and diarrhea several hours after a church
picnic

D 32. A disease that occurs only every once in a while in a particular geographic region

A 33. A disease that is always present in a particular geographical region

*****

A. Cryptosporidium parvum
B. Ebola virus
C. Escherichia coli O157:H7
D. Hantavirus
E. Legionella pneumophila

C 34. The cause of an epidemic associated with hamburgers and a popular fast food
chain

D 35. The cause of the epidemic that occurred in the four corners area of the United
States in 1993

E 36. The cause of the epidemic that occurred in Philadelphia in 1976

A 37. The cause of the epidemic that occurred in Milwaukee in 1993

*****

A. Body louse
B. "Crab" louse
C. Flea
D. Mite
E. Tick

D 38. An arachnid ectoparasite; one species causes human scabies; larval forms of some
species are referred to as "chiggers"

C 39. An insect ectoparasite; the vector of plague

E 40. An arachnid ectoparasite; the vector of Rocky Mountain spotted fever

A 41. Pediculus humanus var. corporus; an infestation with these insects is known as
pediculosis; in the trenches in World War I, these were known as “cooties”

B 42. An insect called Phthirus pubis

E 43. An arachnid ectoparasite; the vector of Lyme disease

CHAPTER 12 QUESTIONS

Chapter 12 Multiple-Choice Questions from Textbook


1. A nosocomial infection is one that ____________________.

a. the patient has at the time of hospital admission


b. is acquired in the community
c. a patient develops during hospitalization or erupts within 14 days of hospital
discharge
d. affects only the nose
e. none of the above

2. An example of a "fomite" would be ____________________.

a. a drinking glass used by a patient


b. bandages from an infected wound
c. a contaminated bedpan
d. soiled bed linens
e. all of the above

3. Which one of the following Gram-positive bacteria is most likely to be the cause of a
nosocomial infection?

a. Streptococcus pneumoniae
b. Staphylococcus aureus
c. Streptococcus pyogenes
d. Clostridium perfringens
e. Clostridium difficile

4. Which one of the following Gram-negative bacteria is least likely to be the cause of a
nosocomial infection?

a. Escherichia coli
b. Pseudomonas aeruginosa
c. an Enterobacter species
d. a Klebsiella species
e. a Salmonella species

5. Protective (reverse) isolation would be appropriate for a ____________________.

a. patient with tuberculosis


b. patient with a diarrheal disease
c. leukopenic patient
d. patient with pneumonic plague
e. patient infected with MRSA

6. Which one of the following is not part of Standard Precautions?

a. handwashing between patient contacts


b. wearing gloves, masks, eye protection, and gowns when appropriate to do so
c. cleaning and reprocessing reusable equipment before it is used for the care of
another patient
d. placing a patient in a private room having negative air pressure
e. proper disposal of needles, scalpels, and other "sharps"

7. A patient suspected of having tuberculosis has been admitted to the hospital. Which one
of the following is not appropriate?

a. Standard Precautions
b. Airborne Precautions
c. Droplet Precautions
d. Source isolation
e. Use of a type N95 respirator by healthcare professional who are caring for the
patient

8. Which one of the following statements about medical asepsis is false?

a. The use of antiseptics is a medical aseptic technique.


b. Disinfection is a medical aseptic technique.
c. Handwashing is a medical aseptic technique.
d. Medical asepsis is considered a clean technique.
e. The goal of medical asepsis is to exclude all microorganisms from an area.

9. Which one of the following statements about source isolation is false?

a. The room is under negative air pressure.


b. Air entering the room is passed through HEPA filters.
c. Transmission-Based Precautions will be necessary.
d. Source isolation is appropriate for patients with tuberculosis, influenza, or
chickenpox.
e. Source isolation is appropriate for patients with meningococcal meningitis,
whooping cough, or influenza.
10. Contact Precautions are required for patients with .....

a. infections due to multidrug-resistant bacteria


b. Clostridium difficile-associated diseases
c. scabies
d. viral hemorrhagic fevers
e. all of the above

Chapter 12 Matching Questions from Student Web Site


A. arthropod-borne infections
B. community-acquired infections
C. iatrogenic infections
D. nosocomial infections
E. nosocomial zoonoses

11. Infections that patients develop while hospitalized or within 14 days of hospital discharge
are considered to be nosocomial infections.

12. Infections that patients have upon hospital admission are considered to be community-
acquired infections.

13. Nosocomial zoonoses may result if healthcare workers fail to wash their hands after
handling laboratory animals or pets being used for pet therapy.

14. Infections that are actually caused by healthcare workers are specifically known as
iatrogenic infections.

15. Each year, in the United States, approximately 2 million hospitalized patients develop
nosocomial infections.

*****

A. Airborne Precautions
B. Contact Precautions
C. Droplet Precautions
D. Medical aseptic technique
E. Surgical aseptic technique

16. In addition to Standard Precautions, Droplet Precautions must be employed when


managing a patient with streptococcal pneumonia.

17. The goal of Surgical aseptic technique is to render and keep objects and areas sterile.

18. In addition to Standard Precautions, Airborne Precautions must be employed when


managing a tuberculosis patient.

19. The goal of Medical aseptic technique is to reduce the number and transmission of
pathogens.

20. In addition to Standard Precautions, Contact Precautions must be employed when


managing a Clostridium-difficile-associated disease.
Chapter 12 True/False Questions from Student Web Site
True 21. Most of the pathogens involved in nosocomial infections come from the patients
themselves.

True 22. An infection that results from urinary catheterization would be considered an
iatrogenic infection.

False 23. Bacteria are the only pathogens that have become drug resistant.

False 24. A patient with tuberculosis should be placed in protective isolation.

True 25. A superinfection with Clostridium difficile could lead to diseases such as
antibiotic-associated diarrhea and pseudomembranous colitis.

True 26. One of the major factors contributing to nosocomial infections is the failure of
healthcare personnel to follow infection control guidelines.

False 27. Joseph Lister is considered the “Father of Handwashing.”

True 28. A leukopenic patient should be placed in a patient room having positive air
pressure.

True 29. Members of a hospital’s Infection Control Committee would investigate


outbreaks/epidemics that occur within that hospital.

False 30. By practicing Standard Precautions, healthcare workers will be protected from
becoming infected, regardless of the type of infectious disease that the patient has.

Additional Chapter 12 Multiple Choice Questions


31. Which one of the following is the major source of the pathogens that cause nosocomial
infections?

a. wet areas in the hospital environment


b. hospital staff members
c. the hospital air conditioning system
d. the patients' own indigenous microflora
e. visitors to the hospital

32. Which one of the following organisms is not one of the four most common causes of
nosocomial infections?

a. Streptococcus pyogenes
b. Enterococcus spp.
c. Escherichia coli
d. Staphylococcus aureus
e. Pseudomonas aeruginosa

33. Which one of the following types of patients should be placed into reverse isolation?

a. a patient with tuberculosis


b. a patient infected with MRSA
c. a leukopenic patient
d. a patient with pneumonic plague
34. __________ infections are always caused by physicians or other health care workers.

a. latent
b. iatrogenic
c. community-acquired
d. nosocomial
e. contagious

35. __________ infections are the most common type of nosocomial infections.

a. skin
b. blood
c. respiratory
d. post-surgical wound
e. urinary tract

CHAPTER 13 QUESTIONS

Chapter 13 Multiple-Choice Questions from Textbook


1. Assuming that a clean-catch midstream urine was processed in the CML, which one of
the following colony counts is the minimum colony count that is indicative of a urinary
tract infection?

a. 1,000 CFU/mL
b. 10,000 CFU/mL
c. 100,000 CFU/mL
d. 1,000,000 CFU/mL
e. 10,000,000 CFU/mL

2. Which one of the following statements is not true about the disk-diffusion method of
antimicrobial susceptibility testing?

a. it is also known as the "Kirby-Bauer test"


b. a pure culture of the organism is required
c. the plate should be incubated in a CO2 incubator for 12 hours
d. the test should be performed in the exact manner described by the NCCLS
e. zone sizes must be interpreted using charts published by the NCCLS

3. Which one of the following statements about cerebrospinal fluid (CSF) specimens is
false?

a. following collection, they should be rushed to the laboratory


b. they are used to diagnose serious conditions such as meningitis and encephalitis
c. they should always be refrigerated
d. they are collected only by physicians
e. they are treated as “stat” (emergency) specimens in the laboratory

4. All of the clinical specimens submitted to the CML must be ____________________.

a. properly selected
b. properly and carefully collected
c. properly transported to the laboratory
d. properly labeled
e. all of the above

5. Which one of the following methods of antimicrobial susceptibility testing is the most
accurate?

a. the agar dilution method


b. the macro broth dilution method
c. the micro broth dilution method
d. the disk-diffusion method
e. the disk-elution method

6. Which one of the following methods of antimicrobial susceptibility testing is the most
popular method in the United States?

a. the agar dilution method


b. the macro broth dilution method
c. the micro broth dilution method
d. the disk-diffusion method
e. the disk-elution method

7. Who is primarily responsible for the quality of specimens submitted to the CML?

a. the pathologist who is in charge of “the lab”


b. the microbiologist who is in charge of the CML
c. the person who collects the specimen
d. the person who transports the specimen to the CML
e. the physician who writes the laboratory request slip

8. Which one of the following is not one of the four major, day-to-day responsibilities of the
CML?

a. process the various clinical specimens that are submitted to the CML
b. process environmental samples
c. isolate pathogens from clinical specimens
d. identify (speciate) pathogens
e. perform antimicrobial susceptibility testing when appropriate to do so

9. Which one of the following sections is least likely to be found in the CML of a small
hospital?

a. Bacteriology Section
b. Immunology Section
c. Mycology Section
d. Parasitology Section
e. Virology Section

10. In the Mycology Section of the CML, molds are identified by ____________________.

a. biochemical test results


b. macroscopic observations
c. microscopic observations
d. a combination of (a), (b), and (c)
e. a combination of (b) and (c)
Chapter 13 Matching Questions from Student Web Site
A. agar dilution method
B. disk diffusion method
C. macro broth dilution method
D. micro broth dilution method

11. Because the macro broth dilution method of antimicrobial susceptibility testing uses
large numbers of test tubes, this method is impractical for use in the Clinical
Microbiology Laboratory.

12. The disk diffusion method of antimicrobial susceptibility testing is also known as the
“Kirby Bauer method.”

13. The agar dilution method of antimicrobial susceptibility testing is considered to be the
“gold standard.”

14. In the United States, the micro broth dilution method is currently the most popular
method for performing of antimicrobial susceptibility testing.

15. Agar plates are used in both the agar dilution method and the disk diffusion method of
antimicrobial susceptibility testing.

*****

A. Bacteriology Section
B. Mycology Section
C. Mycobacteriology Section
D. Parasitology Section
E. Virology Section

16. Mycoses are diagnosed in the Mycology Section of the Clinical Microbiology
Laboratory.

17. Tuberculosis is diagnosed in the Mycobacteriology Section of the Clinical Microbiology


Laboratory.

18. Hair clippings, nail clippings, and skin scrapings are most often processed in the
Mycology Section of the Clinical Microbiology Laboratory.

19. Susceptibility testing is routinely performed only in the Bacteriology Section and the
Mycobacteriology Section of the Clinical Microbiology Laboratory.

20. Miniaturized biochemical test systems – known as “minisystems – are most often used in
the Bacteriology Section of the Clinical Microbiology Laboratory.

Chapter 13 True/False Questions from Student Web Site


True 21. Poor quality clinical specimens are unlikely to produce clinically relevant results.

False 22. The Director of the Clinical Microbiology Laboratory (CML) is ultimately
responsible for the quality of clinical specimens submitted to the CML.

True 23. Special disinfection procedures are required to prevent indigenous microflora of
the skin from contaminating blood cultures.
False 24. Bacteriuria is a sure sign of urinary tract infection.

True 25. CSF specimens are treated as “stat” specimens in the Clinical Microbiology
Laboratory.

False 26. CSF specimens should be refrigerated en route to the Clinical Microbiology
Laboratory.

False 27. There is no need to refrigerate urine specimens for culture if they are clean-catch
midstream specimens.

True 28. Many clinical specimens labeled “sputum” are actually saliva specimens.

True 29. The Clinical Microbiology Laboratory is part of the Clinical Pathology Division
of the Pathology Department.

True 30. Beta-lactamase testing is always performed on isolates of Neisseria gonorrhoeae


and Haemophilus influenzae.

Additional Chapter 13 Multiple Choice Questions


31. Which one of the following is the most common contaminant of blood cultures?

a. Escherichia coli
b. Staphylococcus epidermidis
c. Staphylococcus aureus
d. Streptococcus pneumoniae
e. Streptococcus pyogenes

32. A urine culture was set up using a 0.001 mL calibrated loop. It yielded 100 colonies.
Which of the following results would be reported to the physician?

a. 100 CFU/mL
b. 1000 CFU/mL
c. 10,000 CFU/mL
d. 100,000 CFU/mL

33. Failure to refrigerate a urine culture specimen when a long time will elapse between
specimen collection and delivery to the laboratory is most apt to result in __________.

a. death of the pathogen


b. an abnormally high colony count
c. contamination of the specimen by members of the patient's indigenous microflora
d. an abnormally low WBC count

CHAPTER 14 QUESTIONS

Chapter 14 Multiple-Choice Questions from Textbook


1. Which of the following virulence factors enable(s) bacteria to attach to tissues?
a. neurotoxins
b. endotoxin
c. capsules
d. flagella
e. pili

2. Neurotoxins are produced by ____________________.

a. Staphylococcus aureus and Streptococcus pyogenes


b. Clostridium difficile and Clostridium perfringens
c. Clostridium botulinum and Clostridium tetani
d. Pseudomonas aeruginosa and Mycobacterium tuberculosis
e. both (b) and (c)

3. Which of the following pathogens produce enterotoxins?

a. Staphylococcus aureus and Vibrio cholerae


b. Clostridium difficile and Clostridium perfringens
c. Salmonella spp. and Shigella spp.
d. Bacillus cereus and certain serotypes of E. coli
e. all of the above

4. A bloodstream infection with ____________________ could result in the release of


endotoxin into the bloodstream.

a. Staphylococcus aureus or Streptococcus pyogenes


b. Clostridium difficile or Clostridium perfringens
c. Neisseria gonorrhoeae or Escherichia coli
d. Staphylococcus aureus or Mycobacterium tuberculosis
e. Mycoplasma pneumoniae or Mycobacterium tuberculosis

5. Communicable diseases are most easily transmitted during the __________________.

a. incubation period
b. prodromal period
c. period of illness
d. period of convalescence
e. latent period

6. Enterotoxins affect cells in the ____________________.

a. respiratory tract
b. genitourinary tract
c. gastrointestinal tract
d. central nervous system
e. cardiovascular system

7. Which one of the following Gram-negative bacteria is least likely to be the cause of
septic shock?

a. Pseudomonas aeruginosa
b. Escherichia coli
c. Neisseria meningitidis
d. Mycoplasma pneumoniae
e. Haemophilus influenzae
8. ____________________ produces both a cytotoxin and an enterotoxin.

a. Vibrio cholerae
b. Clostridium difficile
c. Clostridium tetani
d. Clostridium botulinum
. e. Corynebacterium diphtheriae

9. Which of the following virulence factors enable(s) bacteria to avoid phagocytosis by


white blood cells?

a. cell wall
b. cell membrane
c. capsule
d. flagella
e. pili

10. ____________________ can cause toxic shock syndrome.

a. Clostridium difficile and Clostridium perfringens


b. Neisseria gonorrhoeae and Escherichia coli
c. Staphylococcus aureus and Mycobacterium tuberculosis
d. Mycoplasma pneumoniae and Mycobacterium tuberculosis
e. Staphylococcus aureus and Streptococcus pyogenes

Chapter 14 Matching Questions from Student Web Site


A. coagulases
B. enterotoxins
C. kinases
D. necrotizing enzymes
E. neurotoxins

11. The neurotoxins produced by Clostridium botulinum and Clostridium tetani are
examples of virulence factors.

12. The necrotizing enzymes produced by Clostridium perfringens and Streptococcus


pyogenes are examples of virulence factors.

13. Coagulases are exoenzymes that cause clot formation.

14. Kinases are exoenzymes that dissolve clots.

15. Enterotoxins are produced by Clostridium difficile, Salmonella spp., Shigella spp.,
Vibrio cholerae, and certain serotypes of E. coli.

*****

A. adhesins
B. endotoxin
C. hyaluronidase
D. integrins
E. leukocidins

16. Endotoxin is a virulence factor that is found in (and released from) the cell walls of
Gram-negative bacteria.
17. The molecules on the surfaces of host cells that pathogens are able to recognize and
attach to are known as receptors or adhesins.

18. Molecules on the surfaces of pathogens that are able to recognize and bind to molecules
on the surfaces of host cells are known as ligands or integrins.

19. Hyaluronidase is also known as “spreading factor.”

20. Leukocidins are toxins that destroy white blood cells.

Chapter 14 True/False Questions from Student Web Site


True 21. Bacterial capsules protect bacteria from being phagocytized by leukocytes.

False 22. A headache is a classic example of a sign of a disease.

False 23. In order to cause disease, all bacterial pathogens must first attach to some tissue in
the body.

True 24. Rickettsias and chlamydias are examples of obligate intracellular pathogens.

False 25. Babesia spp., Ehrlichia spp., and Plasmodium spp. are examples of
intraerythrocytic pathogens.

False 26. The exoenzyme that causes toxic shock syndrome is called erythrogenic toxin.

False 27. The neurotoxins produced by Clostridium botulinum and Clostridium tetani cause
a spastic, rigid type of paralysis.

True 28. Although most people use the terms “infection” and “infectious disease”
synonymously, microbiologists define infection as colonization by a pathogen.

True 29. Avirulent strains do not cause disease.

True 30. It is thought that the waxes in the cell walls of Mycobacterium tuberculosis
protect this pathogen from digestion within phagocytes.

Additional Chapter 14 Matching Questions


A. Acute disease
B. Asymptomatic disease
C. Chronic disease
D. Latent disease
E. Systemic disease

D 31. Cold sores and shingles are examples

B 32. The same as a subclinical disease

C 33. A disease that has an insidious (slow) onset and lasts a long time

E 34. A disease that has spread throughout the body


A 35. A disease that has a sudden onset, but does not last long

*****

A. Capsules
B. Pili
C. M-protein
D. Kinases
E. Hyaluronidases

B 36. Enable certain strains of Neisseria gonorrhoeae to adhere to the inner walls of the
urethra

A 37. Enable Streptococcus pneumoniae to avoid being phagocytized early in the


infection

C 38. A virulence factor specifically associated with Streptococcus pyogenes

E 39. Enable some bacteria to penetrate deeper and deeper into tissues

D 40. Enable some staphylococci and streptococci to dissolve clots

*****

A. Neurotoxin
B. Enterotoxin
C. Diphtheria toxin
D. Endotoxin
E. Erythrogenic toxin

E 41. Produced by some strains of Streptococcus pyogenes

A 42. Tetanospasmin is an example

D 43. Its presence in the bloodstream can lead to fever and septic shock

A 44. Produced by Clostridium botulinum

B 45. Produced by Clostridium difficile

CHAPTER 15 QUESTIONS

Chapter 15 Multiple-Choice Questions from Textbook


1. Host defense mechanisms – ways in which the body protects itself from pathogens – can
be thought of as an army consisting of ____________________ lines of defense.

a. 2
b. 3
c. 4
d. 5
e. 6
2.. Which of the following is not part of the body’s first line of defense?

a. intact skin
b. mucus
c. perspiration
d. fever
e. the pH of the stomach contents

3. Each of the following is considered a part of the body’s second line of defense except
____________________.

a. fever
b. lysozyme
c. inflammation
d. interferons
e. the complement cascade

4. Which of the following is not a consequence of activation of the complement system?

a. initiation and amplification of inflammation


b. attraction and activation of leukocytes
c. repair of damaged tissue
d. lysis of bacteria and other foreign cells
e. increased phagocytosis by phagocytic cells (opsonization)

5. Each of the following is a primary purpose of the inflammatory response except


____________________.

a. to localize the infection


b. to prevent the spread of microbial invaders
c. to neutralize any toxins being produced at the site
d. to aid in the repair of damaged tissue
e. opsonization

6. Which of the following cells is a granulocyte?

a. monocyte
b. lymphocyte
c. eosinophil
d. macrophage
e. thrombocyte

7. All of the following would be considered an aspect of microbial antagonism except


____________________.

a. competition for nutrients


b. competition for space
c. production of bacteriocins
d. production of antibiotics
e. production of lysozyme

8. Which of the following function as opsonins?

a. antibodies
b. antigens
c. chemotactic agents
d. complement fragments
e. both (a) and (d)

9. Which one of the following statements about interferons is false?

a. Interferons have been used to treat hepatitis C and certain types of cancer.
b. Interferons are virus-specific.
c. Interferons are being produced commercially by genetically-engineered bacteria.
d. Interferons produced by virus-infected rabbit cells cannot be used to treat viral
diseases in humans.
e. The interferons produced by a virus-infected cell will not save that cell from
destruction.

10. Which one of the following in not one of the four cardinal signs or symptoms of
inflammation?

a. edema
b. heat
c. loss of function
d. pain
e. redness

Chapter 15 Matching Questions from Student Web Site


A. bacteriocins
B. chemotactic agents
C. complement fragments
D. cytokines
E. interferons

11. Cytokines are chemical mediators that enable cells to communicate with each other.

12. When attached to the surface of particles or cells, complement fragments can facilitate
phagocytosis.

13. Proteins produced by one bacterial species that kill other bacterial species are known as
bacteriocins.

14. Interferons are small, antiviral proteins that are produced by virus-infected cells.

15. Chemotactic agents attract leukocytes to sites where they are needed.

*****

A. chemotaxis
B. opsonization
C. phagolysosome
D. phagosome
E. vasodilation

16. A phagosome is a membrane-bound vesicle containing only an ingested object.

17. The directed migration of leukocytes is known as chemotaxis.

18. A phagolysosome is a membrane-bound vesicle containing an ingested object and


digestive enzymes.
19. Vasolidation is an increase in the diameter of blood vessels.

20. Opsonization is a process by which phagocytosis is facilitated by the deposition of


antibodies or complement fragments onto the surface of particles or cells.

Chapter 15 True/False Questions from Student Web Site


True 21. Lactoferrin and transferrin are proteins that bind iron, and therefore deprive
pathogens of this essential nutrient.

False 22. Pyrogenic substances cause the production of pus.

False 23. Interferons are virus-specific, but are not species-specific.

False 24. Complement is the name of a single plasma protein that “complements” the
actions of the immune system.

False 25. Eosinophils are much better phagocytes than neutrophils.

True 26. Phagocytes can only ingest objects that they are able to attach to.

False 27. The terms “leukemia” and “leukopenia” both refer to an abnormally low number
of circulating leukocytes.

True 28. Ehrlichia spp. are intraleukocytic pathogens.

True 29. Chemokines are chemotactic agents that are produced by various cells of the
human body.

True 30. Perspiring, swallowing, and urinating are all considered to be part of the first line
of defense.

Additional Chapter 15 Matching Questions


A. Chemotaxis
B. Leukocidin
C. Leukocytosis
D. Leukopenia
E. Vasodilation

D 31. A decreased number of white blood cells in the peripheral bloodstream

E 32. Increase in the diameter of blood vessels; an initial event in inflammation

C 33. An increased number of white blood cells in the peripheral bloodstream

A 34. Considered to be the first phase of phagocytosis; the means by which phagocytic
cells are attracted to sites where they are needed

*****

A. Basophils
B. Eosinophils
C. Macrophages
D. Neutrophils
C 35. Fixed or tissue ____________________ are called histiocytes

A 36. Like mast cells, these cells release histamine and other chemical mediators during
Type I hypersensitivity reactions

B 37. Increased numbers of these cells are found in the peripheral blood stream during
allergic reactions and helminth infections

D 38. These cells are also called polymorphonuclear leukocytes, polys, or PMNs

C 39. These cells are monocytes rather than granulocytes

CHAPTER 16 QUESTIONS

Chapter 16 Multiple-Choice Questions from Textbook


1. Of the following, which is the least likely to be involved in cell-mediated immunity?

a. macrophages
b. antibodies
c. a delayed reaction
d. cytokines
e. TD cells

2. Antibodies are secreted by ____________________.

a. TH cells
b. macrophages
c. basophils
d. plasma cells
e. TC cells

3. Humoral immunity involves all of the following except ____________________.

a. NK cells
b. B cells
c. antibodies
d. plasma cells
e. antigens

4. Immunity that develops as a result of an actual infection is called


____________________.

a. natural passive acquired immunity


b. artificial active acquired immunity
c. natural active acquired immunity
d. artificial passive acquired immunity

5. Artificial passive acquired immunity would result from ____________________.

a. receiving a gamma globulin injection


b. receiving an inactivated vaccine
c. ingestion of colostrum
d. having the measles
e. receiving an attenuated vaccine

6. The vaccines that are used to protect people from diphtheria and tetanus are
____________________.

a. subunit vaccines
b. inactivated vaccines
c. attenuated vaccines
d. toxoids
e. antitoxins

7. Natural passive acquired immunity would result from ____________________.

a. receiving a gamma globulin injection


b. receiving an inactivated vaccine
c. ingestion of colostrum
d. having the measles
e. receiving an attenuated vaccine

8. Which one of the following statements about IgM is false?

a. IgM contains a J-chain.


b. IgM is the first immunoglobulin type produced in the secondary response.
c. IgM has a total of 10 antigen-binding sites.
d. IgM is a pentamer.
e. IgM is a short-lived molecule.

9. Which of the following could be an example or an effect of Type III hypersensitivity?

a. glomerulonephritis
b. rheumatoid arthritis
c. systemic lupus erythematosus
d. immune complex formation
e. all of the above

10. Most likely, immunology got its start in 1890 when these scientists discovered antibodies
while developing a diphtheria toxin:

a. Edward Jenner and Louis Pasteur


b. Emil Behring and Kitasato Shibasaburo
c. James Watson and Francis Crick
d. Jonas Salk and Albert Sabin
e. Elie Metchnikoff and Robert Koch

Chapter 16 Matching Questions from Student Web Site


A. artificial active acquired immunity
B. artificial passive acquired immunity
C. natural active acquired immunity
D. natural passive acquired immunity

11. The immunity that a fetus acquires in utero, as a result of receiving protective antibodies
from Mom’s blood is called natural passive acquired immunity.
12. The immunity that someone acquires as a result of an infection is called natural active
acquired immunity.

13. The immunity that someone acquires after receiving a shot of gamma globulin is called
artificial passive acquired immunity.

14. The immunity that someone acquires as a result of receiving a vaccine is called artificial
active acquired immunity.

15. The immunity that an infant acquires as a result of breast feeding is called natural
passive acquired immunity.

*****

A. antibodies
B. antigens
C. epitopes
D. haptens
E. immune complexes

16. Antigens are also known as immunogens.

17. Molecules referred to as antigenic determinants are also known as epitopes.

18. Antibodies are in a class of proteins called immunoglobulins.

19. Small molecules called haptens are antigenic only when then are coupled with large
carrier molecules such as proteins.

20. Immune complexes initiate Type III hypersensitivity reactions.

Chapter 16 True/False Questions from Student Web Site


True 21. Technically speaking, all antibodies are immunoglobulins, but not all
immunoglobulins are antibodies.

False 22. IgG is the largest of the five classes of immunoglobulins.

True 23. The primary function of NK and K cells is to kill foreign cells, virus-infected
cells, and tumor cells.

True 24. Common allergic reactions, such as those experienced in hay fever, are also
known as anaphylactic reactions.

False 25. IgM antibodies and basophils play major roles in anaphylactic reactions.

True 26. The penicillin molecule is an example of a hapten.

False 27. Autoimmune diseases are always the result of Type II hypersensitivity reactions.

False 28. With respect to a particular pathogen, detection of antibodies in a patient’s blood
provides better proof of current infection than does detection of antigens.

False 29. If a person’s immune system is not functioning properly, that person is said to be
immunocompetent.
True 30. An IgM molecule can bind to ten antigenic determinants, but they would all have
to be the antigenic determinant that stimulated the production of that IgM
molecule.

Additional Chapter 16 Matching Questions


A. Artificial active acquired immunity
B. Artificial passive acquired immunity
C. Natural active acquired immunity
D. Natural passive acquired immunity

C 31. Example: the immunity that results from the body's response to an invading
microorganism

A 32. Example: the immunity that results from the body's response to a vaccine

B 33. Example: antibodies that a person receives in an injection of gamma globulin


(immune serum globulin)

D 34. Examples: maternal antibodies that a fetus receives across the placenta and
maternal antibodies that an infant receives in colostrum

*****

A. B lymphocytes (B cells)
B. Macrophages
C. Mast cells
D. TH lymphocytes (TH cells)
E. TS lymphocytes (TS cells)

A 35. Capable of producing antibody-secreting plasma cells following clone formation

E 36. They help regulate the immune response by turning off certain activities

B 37. Phagocytic cells that act as "antigen-presenting cells" in the immune response to
(processing of) T-dependent antigens

D 38. The major target of human immunodeficiency virus (HIV)

A 39. The only cells required in the immune response to T-independent antigens

*****

A. IgA
B. IgD
C. IgE
D. IgG
E. IgM

D 40. A monomer; the most common type of immunoglobulin; very high levels are
produced in the secondary (anamnestic) response to an antigen

E 41. A pentamer; five monomers linked together by a J-chain; the first type of
immunoglobulin to be formed in the primary response to an antigen

C 42. A monomer; present on the surface of mast cells and basophils


A 43. A dimer; two monomers linked together by a J-chain; frequently found in mucous
membranes

C 44. Produced by atopic individuals in response to allergens

*****

A. Type I hypersensitivity reactions


B. Type II hypersensitivity reactions
C. Type III hypersensitivity reactions
D. Type IV hypersensitivity reactions

C 45. Immune complex reactions; antigen-antibody complexes become deposited in


tissue (e.g., the glomeruli of the kidneys), resulting in inflammatory damage

A 46. Anaphylactic reactions; examples include hay fever symptoms, hives, asthma, and
the anaphylactic shock that could follow a bee sting or administration of penicillin

D 47. Cell-mediated reactions; antibodies don't play a major role

B 48. Cytotoxic reactions; involve the lysis of cells; the adverse reactions that follow
incompatible blood transfusions are an example

CHAPTER 17 QUESTIONS

Chapter 17 Multiple-Choice Questions from Textbook


1. The most common sexually transmitted disease in the U.S. is caused by
____________________.

a. Escherichia coli
b. Neisseria gonorrhoeae
c. Chlamydia trachomatis
d. Candida albicans
e. Trichomonas vaginalis

2. Infectious hepatitis is caused by ____________________.

a. HAV
b. HBV
c. HCV
d. HDV
e. HEV

3. Streptococcus pneumoniae is a common cause of ____________________.

a. otitis media
b. pneumonia
c. meningitis
d. cold sores
e. all of the above except (d)
4. Staphylococcus aureus is a common cause of ____________________.

a. toxic shock syndrome


b. food poisoning
c. nosocomial infections
d. skin and wound infections
e. all of the above

5. Which of the following diseases may be caused by Streptococcus pyogenes?

a. strep throat
b. necrotizing fasciitis
c. toxic shock syndrome
d. impetigo
e. all of the above

6. Which of the following diseases may be caused by Chlamydia trachomatis?

a. non-gonococcal urethritis (NGU)


b. trachoma
c. inclusion conjunctivitis
d. lymphogranuloma venereum (LGV)
e. all of the above

7. An infection of the urinary bladder is known as ____________________.

a. urethritis
b. ureteritis
c. cystitis
d. nephritis
e. pyelonephritis

8. Which of the following organisms is the most common cause of urethritis?

a. Escherichia coli
b. Neisseria gonorrhoeae
c. Chlamydia trachomatis
d. Candida albicans
e. Trichomonas vaginalis

9. Which of the following organisms is the most common cause of cystitis?

a. Escherichia coli
b. Neisseria gonorrhoeae
c. Chlamydia trachomatis
d. Candida albicans
e. Trichomonas vaginalis

10. Which of the following associations is incorrect?

a. Lyme disease ... tick


b. plague ... rat flea
c. Rocky Mountain spotted fever ... tick
d. West Nile virus encephalitis ... mosquito
e. cryptococcosis ... parrots and parakeets

Chapter 17 Matching Questions from Student Web Site


A. Staphylococcus aureus
B. Staphylococcus epidermidis
C. Streptococcus agalactiae
D. Streptococcus pneumoniae
E. Streptococcus pyogenes

11. Staphylococcus epidermidis is a common cause of subacute bacterial endocarditis (SBE),


and a less common cause of cystitis.

12. Certain strains of Streptococcus pyogenes are the so-called “flesh-eating bacteria.”

13. Streptococcus pneumoniae is a common cause of bacterial pneumonia, meningitis, and


otitis media.

14. Staphylococcus aureus is the most common cause of toxic shock syndrome.

15. Streptococcus agalactiae is one of the most common causes of neonatal meningitis.

*****

A. Chlamydia species
B. mold
C Rickettsia species
D. virus
E. yeast

16. A tinea infection is caused by a mold.

17. Trachoma is caused by a Chlamydia species.

18. Cryptococcosis is caused by a yeast.

19. Any type of wart is caused by a virus.

20. The most common cause of non-gonococcal urethritis is a Chlamydia species.

Chapter 17 True/False Questions from Student Web Site


False 21. Tuberculosis and Hansen’s disease (leprosy) are caused by Mycoplasma species.

True 22. It is possible for scarlet fever and toxic shock syndrome to be caused by the same
pathogen.

True 23. The most common cause of gas gangrene also causes a type of food poisoning.

False 24. The diseases caused by Mycobacterium species are referred to as mycoses.

True 25. Botulism, gas gangrene, and tetanus are all caused by Clostridium species.

True 26. The etiologic agent of infectious mononucleosis also causes or is associated with
various types of human cancers.

False 27. Measles, German measles, mumps, and whooping cough are all caused by
viruses.
True 28. Alpha-hemolytic streptococci of oral origin and Staphylococcus epidermidis are
common causes of subacute bacterial endocarditis (SBE).

False 29. Haemophilus influenzae is the most common cause of influenza (“flu”).

True 30. Meningitis could be caused by a virus, a bacterium, a fungus, or a protozoan.

Additional Chapter 17 Matching Questions


A. Candida albicans
B. Chlamydia trachomatis
C. Neisseria gonorrhoeae
D. Escherichia coli
E. Treponema pallidum

C 31. This Gram-negative diplococcus can cause pelvic inflammatory disease (PID),
endocarditis, meningitis, arthritis, pharyngitis, skin rash, and proctitis, in addition
to other diseases; it is a major cause of urethritis

E 32. This spirochete cannot be grown on artificial media

B 33. Certain serotypes of this Gram negative bacterium are the major cause of NGU

B 34. An obligate intracellular pathogen

A 35. Causes approximately one-third of the cases of vaginitis

C 36. The cause of ophthalmia neonatorum

*****

A. Candida albicans
B. Coagulase-negative Staphylococcus spp.
C. Pseudomonas spp.
D. Staphylococcus aureus
E. Streptococcus pyogenes

D 37. Gram-positive cocci that are often associated with pimples, sties, folliculitis,
furuncles, carbuncles, impetigo of the newborn, and scalded skin syndrome

E 38 Gram-positive cocci that may be associated with scarlet fever, impetigo,


erysipelas, and glomerulonephritis

C 39. Gram-negative bacilli that may be associated with dermatitis, otitis externa,
"swimmer's ear," and burn wound infections

E 40. Certain strains of this organism are the "flesh-eating" bacteria

D 41. The most common cause of toxic shock syndrome

*****

A. Herpes simplex
B. Epstein-Barr
C. Rubella
D. Rubeola
E. Shingles

E 42. Follows reactivation of the virus that causes chickenpox

A 43. Associated with cold sores, fever blisters, and genital lesions; can also cause eye
infections and encephalitis

C 44. The virus that causes German measles

B 45. The cause of infectious mononucleosis; this virus is also known to be oncogenic

*****

A. Ehrlichia spp.
B. Chlamydia trachomatis
C. Haemophilus aegyptius
D. Neisseria gonorrhoeae
E. Mycobacterium tuberculosis

C 46. Gram-negative bacilli that are a common cause of conjunctivitis ("pink eye")

B 47. Some serotypes of this organism cause inclusion conjunctivitis, whereas other
serotypes are the leading cause of blindness in the world

A 48. Tickborne intraleukocytic pathogens

D 49. Eyes of newborn infants are flushed with 1% silver nitrate or antibiotic solutions
to prevent disease due to these bacteria

*****

A. Cystitis
B. Encephalitis
C. Keratitis
D. Meningitis
E. Myelitis

A 50. Infection/inflammation of the urinary bladder

E 51. Infection/inflammation of the spinal cord

B 52. Infection/inflammation of the brain

D 53. Infection/inflammation of the membranes surrounding the brain

*****

A. Group B, beta-hemolytic streptococcus (Streptococcus agalactiae)


B. Haemophilus influenzae
C. Listeria monocytogenes
D. Neisseria meningitidis
E. Streptococcus pneumoniae

E 54. Gram-positive diplococci that are one of the three major causes of bacterial
meningitis; frequently associated with meningitis in elderly patients
A 55. Gram-positive cocci that are one of the three major causes of neonatal meningitis

D 56. Gram-negative diplococci that are one of the three major causes of bacterial
meningitis

B 57. Gram-negative bacilli that are one of the three major causes of bacterial
meningitis; most often associated with meningitis in young children

C 58. Gram-positive bacilli that are one of the three major causes of neonatal meningi-
tis; humans frequently become infected following ingestion of contaminated dairy
products

*****

A. Clostridium difficile
B. Clostridium botulinum
C. Clostridium perfringens
D. Clostridium tetani

C 59. The major cause of gas gangrene (myonecrosis)

B 60. An intoxication due to toxins produced by this anaerobic, sporeforming, Gram-


positive bacillus can lead to a flaccid type of paralysis and death

A 61. An anaerobic, sporeforming, Gram-positive bacillus that is sometimes present as


part of a person's normal intestinal flora; can overgrow when that person takes
oral antibiotics, leading to conditions such as antibiotic-associated diarrhea and/or
pseudomembranous colitis

D 62. An infection with this anaerobic, sporeforming, Gram-positive bacillus can lead to
a rigid type of paralysis and death

C 63. An anaerobic, sporeforming, Gram-positive bacillus that is a common cause of


relatively mild food poisoning

*****

A. Tuberculosis
B. Leprosy
C. Plague
D. Anthrax
E. Tularemia

D 64. Caused by a sporeforming, Gram-positive bacillus

B 65. Caused by acid-fast bacilli that cannot be cultivated on artificial media

A 66. Caused by slow-growing, acid-fast bacilli that can be cultivated on artificial


media

C 67. A flea-borne disease caused by Yersinia pestis

*****

A. Endocarditis
B. Myocarditis
C. Pericarditis
D. Septicemia
E. Ischemia

C 68. Infection/inflammation of the sac around the heart

B 69. Infection/inflammation of the heart muscle tissue

A 70. Infection/inflammation of the inner wall of the heart and heart valves

E 71. Necrosis due to __________ is called gangrene

*****

A. Borrelia
B. Ehrlichia
C. Chlamydia
D. Neisseria
E. Rickettsia

A 72. Species of this spirochete cause tick-borne relapsing fever and Lyme disease

C 73. A species in this genus causes the most common nationally notifiable disease in
the United States

E 74. Species of this obligate intracellular pathogen cause typhus and typhus-like
diseases

*****

A. Lymphadenitis
B. Lymphadenopathy
C. Lymphangitis
D. Lymphokine
E. Lymphoma

C 75. Inflammation of lymphatic vessels

A 76. Inflammation of lymph nodes

B 77. Diseased lymph nodes

*****

A. Enterotoxigenic E. coli (ETEC)


B. Enterohemorrhagic E. coli (EHEC)
C. Salmonella
D. Shigella
E. Staphylococcus aureus

E 78. Food poisoning due to these Gram-positive cocci is actually an intoxication, in


that humans ingest food containing enterotoxins that are produced in vitro

A 79. The most common cause of "traveler's diarrhea" (diarrhea that occurs in tourists)

B 80. The organism involved in the epidemic that was associated with contaminated
hamburger meat
C 81. Infection with these Gram-negative bacilli is most closely linked to contaminated
chickens, turkeys, or eggs

C 82. "Typhoid Mary" was an asymptomatic carrier of these Gram-negative bacilli

CHAPTER 18 QUESTIONS

Chapter 18 Multiple-Choice Questions from Textbook


1. Humans develop malaria following the injection of Plasmodium ____________________
into the bloodstream by an infected female Anopheles mosquito when she takes a blood
meal

a. male and female gametocytes


b. merozoites
c. schizonts
d. sporozoites
e. trophozoites

2. These Plasmodium life cycle stages must be ingested by a female Anopheles mosquito in
order for the Plasmodium life cycle to continue in the mosquito.

a. male and female gametocytes


b. merozoites
c. schizonts
d. sporozoites
e. trophozoites

3. Which of the following protozoal diseases is not transmitted via an arthropod vector?

a. African trypanosomiasis
b. American trypanosomiasis
c. babesiosis
d. giardiasis
e. leishmaniasis

4. Which of the following protozoal diseases is least likely to be transmitted via blood
transfusion?

a. African trypanosomiasis
b. American trypanosomiasis
c. babesiosis
d. malaria
e. trichomoniasis

5. Which of the following protozoal diseases is least likely to be transmitted via an infected
food handler who fails to wash his/her hands after using the bathroom?

a. amebiasis
b. cryptosporidiosis
c. cyclosporiasis
d. giardiasis
e. toxoplasmosis
6. You are visiting a friend whose parents raise pigs. Which of the following diseases are
you most likely to acquire by drinking well water at their farm?

a. amebiasis
b. balantidiasis
c. cryptosporidiosis
d. cyclosporiasis
e. giardiasis

7. You are working on a cattle ranch. Which of the following diseases are you most apt to
acquire as you perform your duties at the ranch?

a. amebiasis
b. balantidiasis
c. cryptosporidiosis
d. cyclosporiasis
e. giardiasis

8. Which of the following protozoal diseases are you most likely to acquire by eating a rare
hamburger?

a. amebiasis
b. balantidiasis
c. cryptosporidiosis
d. giardiasis
e. toxoplasmosis

9. Which of the following associations is incorrect?

a. African trypanosomiasis ... tsetse fly


b. amebiasis ... fecally-contaminated water
c. Chagas’ disease ... mosquito
d. leishmaniasis ... sandfly
e. toxoplasmosis ... cats

10. Which one of the following is an example of an infectious disease that is caused by a
facultative parasite?

a. African trypanosomiasis
b. babesiosis
c. giardiasis
d. malaria
e. primary amebic meningo encephalitis

Chapter 18 Matching Questions from Student Web Site


A. Ciliata
B. Mastigophora
C. Sarcodina
D. Sporozoa

11. African trypanosomiasis, American trypanosomiasis, giardiasis, and trichomoniasis are


caused by protozoa in the category of protozoa known as Mastigophora.

12. Protozoa in the category of protozoa known as Sarcodina move by means of


pseudopodia.
13. Cryptosporidiosis, malaria, and toxoplasmosis are caused by protozoa in the category of
protozoa known as Sporozoa.

14. In the category of protozoa known as Ciliata, only one protozoan causes human disease.

15. A disease known as PAM is caused by a protozoan in the category of protozoa known as
Sarcodina.

*****

A. oocyst
B. ookinete
C. schizont
D. trophozoite
E. zygote

16. In the malarial parasite’s life cycle, a schizont contains numerous merozoites.

17. In the malarial parasite’s life cycle, sporozoites are produced within an oocyst.

18. In the malarial parasite’s life cycle, an ookinete is a motile organism within the
mosquito’s stomach.

19. In the malarial parasite’s life cycle, a/an trophozoite may mature to become a female
gametocyte, a male gametocyte, or a schizont.

20. In the malarial parasite’s life cycle, the oocyst is located on the outer wall of the
mosquito’s stomach.

Chapter 18 True/False Questions from Student Web Site


True 21. In a parasite’s life cycle, the definitive host harbors the adult or sexual stage of the
parasite or the sexual phase of the life cycle.

False 22. The amebae that cause amebic keratitis and PAM are good examples of obligate
parasites.

False 23. In the malarial parasite’s life cycle, humans serve as definitive hosts.

True 24. When causing infections, parasitic protozoa and helminths are endoparasites.

False 25. Scabies is caused by an insect.

True 26. In a parasite’s life cycle, it is possible for a particular arthropod to serve as both a
host and a biological vector.

True 27. It is possible to acquire cryptosporidiosis, cyclosporiasis, and toxoplasmosis by


ingesting oocysts.

True 28. Trichomonas vaginalis cannot survive very long outside of the human body
because it has no cyst form.

True 29. Toxoplasmosis could be acquired by eating raw or rare meat.

False 30. African trypanosomiasis and American trypanosomiasis are transmitted by the
same type of arthropod vector.

Additional Chapter 18 Matching Questions


A. Macroscopic examination of a fecal specimen
B. Microscopic examination of a fecal specimen
C. Microscopic examination of a "Scotch tape" preparation
D. Microscopic examination of a stained peripheral blood smear
E. Microscopic examination of a urogenital discharge or secretion

D 31. The usual way in which African and American trypanosomiasis are diagnosed

B 32. The usual way in which giardiasis is diagnosed

D 33. The usual way in which malaria is diagnosed

E 34. The usual way in which trichomoniasis is diagnosed

B 35. The usual way in which amebic dysentery is diagnosed

*****

A. A mosquito
B. A type of fly
C. Mite
D. Reduviid bug
E. Tick

B 36. African trypanosomiasis

D 37. Chagas' disease

E 38. Babesiosis

D 39. American trypanosomiasis

A 40. Malaria

*****

A. Gametocytes, schizonts, and trophozoites


B. Microfilariae
C. Trophozoites and/or cysts
D. Trypomastigotes
E. Eggs and larvae

D 41. Might be observed in a microscopically-examined, stained, peripheral blood


smear from a patient who has African sleeping sickness

C 42. Might be observed in a microscopically-examined, stained, fecal preparation from


a patient who has giardiasis

A 43. Might be observed in a microscopically-examined, stained, peripheral blood


smear from a patient who has malaria
C 44. Might be observed in a microscopically-examined, stained, fecal preparation from
a patient who has amebic dysentery

D 45. Might be observed in a microscopically-examined, stained, peripheral blood


smear from a patient who has Chagas’ disease

*****

A. Ingestion of cysts in contaminated drinking water


B. Ingestion of oocysts in contaminated water
C. Ingestion of oocysts from cat feces or cysts in contaminated meat
D. Direct (usually sexual) contact with an infected individual
E. Injection of sporozoites by an Anopheles mosquito

C 46. The usual way in which toxoplasmosis is acquired

A 47. The usual way in which giardiasis is acquired

B 48. The usual way in which cryptosporidiosis is acquired

D 49. The usual way in which trichomoniasis is acquired

E 50. The usual way in which malaria is acquired

You might also like